AMA | July 2021

Welcome to the July 2021 Ask Me Anything episode of Mindscape! These monthly excursions are funded by Patreon supporters (who are also the ones asking the questions). I take the large number of questions asked by Patreons, whittle them down to a more manageable size — based primarily on whether I have anything interesting to say about them, not whether the questions themselves are good — and sometimes group them together if they are about a similar topic. Enjoy!

Support Mindscape on Patreon.

[accordion clicktoclose=”true”][accordion-item tag=”p” state=closed title=”Click to Show AMA Questions Only”]Click above to close. Eric Chen What’s your take on Pascal’s wager? Do you think there is a simple explanation for why the argument fails, and if so, do you think you have a satisfactory account of how to make decisions/do ethics when infinities are at stake? Daniel Westwater Given there’s more evidence for galaxies without dark matter. Do you think it would be possible for the universe to exist without it and what would it look like if so? Andrei Dinu In your latest podcast with John Preskill you touch upon the possibility that physical space and distances between objects are emergent phenomena arising from quantum entanglement. You say the following: it’s not the entanglement between you and me that gives us the distance, it’s the entanglement between invisible degrees of freedom in the space between us, which is a little bit hard to visualize. Can you expand on this? Richard Riley You told listeners a couple of months ago that you won’t be renewing your research professor contract at Caltech, and will focus instead on your podcasts/Patreon/YouTube work and your other writing plus your Santa Fe Institute activity. Are you concerned that not being fully in the mix at a top research university will make you drift away from the center of the physics/cosmology world, no matter what else you may be doing? **************************art Anita Tomasik I wonder if there is any tendency among theoretical physicists to prefer abstract art to figurative art? Chris What are your views on art? What is it? What is its purpose? How does it fit into your worldview? Is there anything deep about it or is it just a pleasurable activity like eating a tasty meal? Lana Salameh Who are some of your favourite artists out there? Whether it was in the fine arts, music, film, literature… even comedy! ****************************** Manuel Bevand In a 2014 world science festival debate on quantum mechanics, you said something akin to « if JRW turned out to be true I would probably retire from physics ». Do you still think that today? If yes, could you elaborate on why a little more than you did at the time? Mikolaj Szabo I’ve recently come across an old Nautilus article, that contrasted Neil deGrasse Tyson and Sam Harris, as representatives of two very different but characteristic attitudes of public intellectuals. One is the very opinionated, “the edgier the better” stance, and the other could be called the “educator”, never taking sides or definite positions. My question is what do you think about the role and responsibilities of public thinkers in our society? And do you think there really is such a spectrum, and if so, where would you ideally want to be on it, and where do you think you actually are on this spectrum? JOHN PERRY Taking on board strands of various podcasts, what chances do you give to Democracy winning out as the “optimum” model for national government, given the increasing economic dominance and military power of countries like China, and the level of digital monitoring of individuals/ organisations etc they are rapidly being able to achieve? The ability for bureaucratic control has never been greater. How can we maintain our individual freedoms and truly compete? John T Many podcasts often include scientists as guests to speak on controversial topics. But “scientist” is vague. Anyone can claim to be a scientist. So, as a lay person, listening to podcasts with scientist guests, how can I distinguish their credibility to speak with genuine authority? What heuristics can we use to weight these opinions and protect ourselves against the fallacy of appeals to authority? Alexander Cordova When discussing quantum mechanics, the example that is always used is the electron. However, it’s also stated that everything is quantum in nature at a small enough scale. Do protons/neutrons have wave functions as well? Paul Cousin Thibaut Giraud, a French YouTuber, recently did an experiment on his philosophy channel “Monsieur Phi”. He surveyed twelve thousand of his viewers with carefully crafted thought experiments about free will and morality. As an example of an interesting and somewhat surprising result, around 80% were intuitive compatibilists. What do you think about the idea of experimental philosophy? Connor Scott I know the Schrodinger equation doesn’t predict wavefunction collapse, but does it definitively predict wavefunction branching, in the case where there is no collapse? Is that something that everyone can agree on? Julian Reif Carlo Rovelli wrote the following about the Many Worlds interpretation in his new book, “Helgoland”: “It is not enough to know the ψ wave and Schrödinger’s equation in order to define and use quantum theory: we need to specify an algebra of observables, otherwise we cannot calculate anything and there is no relation with the phenomena of our experience. The role of this algebra of observables, which is extremely clear in other interpretations, is not at all clear in the Many Worlds interpretation.” What do you think about this criticism? Richard McLay I recently listened to a somewhat frustrating podcast on Post Modernism and Post Truth. What are your thoughts on the existence of objective truth? Casey Mahone Given that all branches of the wavefunction are equally real, there would presumably be some bizarre “unexplainable” things happening in some branches. A car passing through a wall, an elephant appearing out of thin air etc… Given the enormous number of times the wavefunction branches each second, is it possible that some of these freaky events have occurred in the history of our own branch, thus accounting for some ghost stories or other bizarre phenomena? David Franke If you were Daryl Morey, what do you do with Ben Simmons this offseason? *************************expanding space Nuzzley-Fuzzily Torsten Pihl of The Satanic Temple It’s my understanding that the universe has a kind of resoulution of plank size. As the universe expands, is the plank size ever so slightly increasing too, or is the universe increasing the amount of, to put it in a cutsie way, “plankies”? William Garrett Since matter, including ourselves, is mostly “empty” space, and the universe seems to be expanding, are we expanding with it? Is subatomic space expanding similarly to “outer” space? Paul Hardie I read an article suggesting that inflation theory might mean that space has been expanding forever. I’ve also heard that space might not be fundamental. Are those two ideas contradictory? *************************** **************************entropy Ken Clowes You have demonstrated entropy by mixing cream and coffee. There are interesting swirls before we reach maximum entropy of bland beige uniformity. But I tried the experiment and about a day later the cream had risen to the top: the mixture had been “unmixed”. Richard Graff My understanding from discussions on Mindscape and your Greatest Ideas series is that there are two definitions of entropy: one subjective that measures an observer’s knowledge of a system, and one objective that measures the disorder of a system. How are these two definitions related? ************************************ *****************************entropy 2 Travis Hinck PBS Spacetime just did a video explaining how the *growth* of quantum entanglement drives the inexorable increase in entropy we see in the macro world (2nd law). My question is: why was there an initial state where particles were not maximally entangled? Did everything start out un-entangled at the Big Bang? Why? Niclas Wiberg Is there a close relation between quantum decoherence and the increase of entropy? Both seem to be related to the arrow of time, and both seem to involve statistical system descriptions. Michele Vincenzi Did the big bang ” create ” the multiverse or was it already going?, if it was already going would be right to assume that we got differents big bang in each universe or even universe with no big bang at all or some completely different process? thanks a lot for your works and podcasts. Kathi Seeger In your conversation on the podcast with Max Tegmark you both agreed on giving more credence to an Everettian multiverse than to a cosmological multiverse to exist. I wondered if, say when the Big Bang started off, the universal wave function also kicked in right from the start? Wouldn’t it be likely that the branching already affected very very early states of the evolving universe and therefore in the process creating multiverses in the “cosmological sense”? ******************************* *******************************entropy 3 Nick G Where does entropy and the 2nd law of thermodynamics fit in the fundamental-ness hierarchy? Is it just an emergent phenomena that isn’t exactly written into the “source code” of the universe, or is it something deeper and more fundamental? Paul Torek Nick G’s question about where entropy fits in the fundamental-ness hierarchy inspires my question: do we even *need* a fundamental-ness hierarchy? Or is it just a matter of which order of presentation of facts we find explanatory, and/or which statements of “laws” and “boundary conditions” are simpler? Or is the latter all that is meant by “fundamental”? ******************************* Allison Could Krasnikov tubes (permanent superluminal tunnels), in principle, be used to construct galaxy-sized telescopes? Michael Lesniak I first became familiar with you on Science Channel shows like “Through the Wormhole”, and I’m curious: how did the interview process for shows like that go (who contacted whom, did they do one long interview or separate ones for each episode, etc.) and how come I haven’t seen you on any more recent shows like that? Paul Hess Is a nation of millions of people “conscious”? Jeff B PRIORITY QUESTION! The question boils down to how a waving field ends up getting detected as a dot on a detector screen (rather than some other shape). Your previous answer was basically “local interactions plus decoherence”. This makes sense to me, but only under the assumption that dots are somehow given special treatment during wavefunction branching. Otherwise I could imagine the wavefunction branching so that each branch saw something other than a dot appear on the screen (a circle, triangle, whatever). My understanding is that these dots are special because they are “pointer states” that are stable under decoherence. So my question becomes a bit meta…WHY are these pointer states and not something else? Our world would probably look very different if the wavefunction branched differently, but it seems that there has got to be some underlying ontological reason for why it branches in this particular way. Claudio Slamovits I read a biography of Fred Hoyle, and he strikes me as a fascinating figure. He seems to have made important contributions, especially on nucleosynthesis. He also had controversial, later debunked views such as the steady-state (i.e., no Big Bang) universe. How is Hoyle perceived in the Physics world? An underrated genius or a crackpot? Peter Solfest After re-reading The Big Picture, I was wondering about strong emergence and QFT. Is it theoretically possible/mathematically consistent to construct a quantum field which only interacts with other fields if they are sufficiently complex? Andrew J Laplace’s demon has perfect knowledge of all microstates for a single instant in time as well as a full understanding of the true laws of physics and can therefore predict the future and retrodict the past. Is the demon then limited to predictions/computations only for microstates? If that’s the case, then isn’t the demon essentially clueless about predicting any future that might be meaningful to people? Orin Harris In your last AMA you invoked Lewis’ possible worlds to explain your position on the PSR, saying “the fact that we live in one [world] rather than the other has got to come down to a brute fact”. I found your response surprising because Lewis’ Modal Realism provides a reason for apparent brute facts, and that reason (self-location uncertainty) is the very same invoked by yourself in answers to “why do we find ourselves in this branch rather than the other” in Everettian QM. The anthropics that provide the “reason” for apparent brute facts in the MWI are the very same mechanism by which brute facts can be explained in Lewis’ model, which is an example of a metaphysics that at bottom satisfies the PSR, by being maximally symmetric, not reifying one brute fact over any other. Can you expand on why you don’t accept brute fact explanations of chancy outcomes in QM, but do accept them in a seemingly similar situation? Suraj Rajan Universe is amoral and the prospect of a “no-consequence after death” could be either liberating or disappointing. Liberating as in, this is the one life we get to live, so live it as you please and satisfy your brain’s whims and fancies. Disappointing as in, no matter what legacy or “goodness” I leave after my death, it is not going to be of any use to me, I’ll be non-existent. Do you have a different take on this matter? Sam Hartzog If memory serves, I was introduced to Robert L. Forward’s “Dragon’s Egg” by your recommendation on some forum or another. If so, thanks! I LOVE this book. – Have modern field theories ruled out the complex, ultra-fast, nucleonic interactions that give rise to the neutron star’s “chemistry”? Jim Murphy You have said before that you are convinced space is not fundamental, but are less convinced about time. How can this be the case if space and time are just two aspects of the same thing? Alex Borland I have a question about the simulation hypothesis. Imagine that humanity managed to simulate a universe complex enough to contain intelligent life. If that intelligent life started to study their universe and try to discover its physics, at what point would you give them credit and say they had cracked its “fundamental” laws? Would it be once they had figured out all the equations our supercomputer was running? Or would they need to discover a “deeper” level of reality such as the programming language they were written in, or the hardware they were running on, etc. krathorlucca last month in your AMA you mentioned that eventually our knowledge will bottom out. I interpreted this as you saying that eventually progress stops because we reach some ‘cold hard facts’. I agree there probably is a place where things bottom out, namely reality. However, how can we ever hope to know we’ve reached such a place? **********************************podcast Jesse Rimler I’m continually impressed by the variety (and quality) of your guests. Could you tell us a little about your methods of discovery? As an author, do you receive advanced copies of books? Russell Wolf In a recent ama you talked about how you always take recommendations for podcast guests but don’t like to make any promise about whether they’ll appear. I was wondering if you could say anything in aggregate about how often a listener recommendation results in a guest that you otherwise weren’t considering. Seth Would you ever consider a podcast where you go into detail about your day to day process for producing a podcast? I really love how accessible you seem and how you don’t make it seem like what you do is magic. ********************************** Gregory Mendell PRIORITY QUESTION: Consider: you measure an electron’s spin along axis “a” in St. Louis. Spin up, you go to Los Angeles (LA), spin down, you go to New York (NY). Your spouse subsequently measures the spin of the same electron along axis “b”, and goes to Minneapolis (MN) if spin up and to New Orleans (NO) if spin down. If a = b, I think you and your spouse are in the state |Psi) = 1/sqrt(2) |LA,MN) + 1/sqrt(2) |NY,NO). Your best friend calls you and observes your location. Your spouse’s best friend calls and observes your spouse’s location. The friends correlate their results and repeat for axes combinations (yours and your spouses) of (a,b), (a,c) and (b,c). Question: Is Bell’s Inequality violated as per QM, proving you (and your spouse) were in two places at the same time, or not? Piu PRIORITY QUESTION: If you could ask an all knowing entity any 3 questions about the universe(s) we live in, what would they be? Greg Fife At this time, what’s your Bayesian assessment of the possibility that the Fermilab G-2 result actually demonstrates new physics? If it does turn out to be new physics, do you have any view on what that will most likely turn out to be? ***************************** fields Cooper Quanta Magazine ran an article about the results of the recent Muon g-2 experiment which included this quote: “The particle you thought was a bare muon is actually a muon plus a cloud of other things that appear spontaneously.” I’ve always imagined the quantum foam as something that exists everywhere regardless of the local presence of any “real” particles. This quote, however, makes it sound like virtual particles only exist in clouds around “real” particles. Is the quantum foam something that particles “travel through”? Or is it something that is created by the local presence of a particle? Peter Whaley It’s my understanding that particles like photons are perturbations in the electromagnetic field. My question is what makes the disturbance in the field? If it’s particles, what came first the chicken or the field? ************************************* Anti-de Shitter Space PRIORITY QUESTION In your podcast with Max Tegmark, he proposes the Mathematical Multiverse in which our universe is just one of many mathematical structures, all of which are equally real. As a non-physicist, I often wonder how ideas like this arise. When a physicist proposes an idea like this does it originate from some physical intuition, an attempt to be original, or something else? What credence should we or can we assign to theories that are not testable? Eli Graham PRIORITY QUESTION: It seems to me that quantum physics and (the philosophical movement of) phenomenology–Husserl, Heidegger, Merleau-Ponty, etc,–appeared and developed at about the same time, and in the same intellectual climate. Do you think quantum physics and phenomenology influenced each other? anonymous Are you optimistic or pessimistic that the USA, and the world, will meet the required greenhouse gas reduction targets in time (and stay within other identified ecological thresholds)? If pessimistic, do you sometimes think about whether you, as an exceptionally smart person with a large network and large audience, could do more to steer it in the right direction, e.g. with your choice of guests or the questions you ask, by steering public debate, spreading new insights into people’s and thought leaders’ minds,…? Charles Hertz PRIORITY QUESTION: One of the touted benefits of the concept of inflation is that it solves the “horizon problem.” I know some people (e.g. Roger Penrose) don’t believe in inflation, and I think that you have said in the past that you are 50:50 on it. Suppose evidence, empirical or theoretical, is found that clearly, to your satisfaction, completely discredits the concept of inflation. Can you speculate on what other kinds of processes or events might then need to be brought into play to solve the horizon problem? Sandro Stucki My question is about Mach’s principle. I’ve heard it stated as something like “the large-scale distribution of mass influences your rotational reference frame” or “mass out there influences inertia here”. These don’t make sense to me. It seems to me that even if there were no stars and planets out there, I could still feel the difference between myself spinning or remaining at rest. Spinning me would experience a centrifugal force even in the absence of “mass out there”, no? I feel like I must be missing something. Could you shed some light on this? **************************free will Siddhartha Are libertarian free will and a block universe mutually exclusive? We can only have libertarian free will if the future is yet to be determined. While in a block universe, the future is already set in (probabilistic) stone. Are these ideas sensible? Redbeard What distinguishes the block universe from someone at the end of time with knowledge of history? I ask because one has nothing to say about fate or free will, and the other has been interpreted to preclude free will. Is there a philosophical difference between me knowing my history at the end of my life and the block universe view of my life? **************************** Alas, Poor Yorick The connections between and physics and philosophy are fascinating and it feels to me fairly crucial to a scientific temperament, yet my physicist colleagues seem to relish in a wholesale rejection of the entire broad tradition of philosophy. I’m often trying to get these friends to take philosophy beyond a vague impression of Popper, and politics other than naive technocracy seriously. I was wondering if you had any thoughts on where this antagonism comes from and how to counter it, as it seems quite pervasive and popular among famous scientists who should know better. Frederik Kloch Lorits I’ve heard you mention that Many Worlds doesn’t have any philosophical implications, but how about a case where a person is given the choice to sacrifice themselves to save humanity? If the person believes that this universe is all there is, they would be inclined to sacrifice themselves, because the alternative is the end of humanity in a final and absolute sense. But if they knew that there was a near-infinite amount of other worlds where humanity would continue on, then the person wouldn’t have to worry as much about destroying humanity, because this is just one world out of many. hilbertspaceman What would constitute experimental evidence that Schrodinger’s equation does not always hold? For example, why isn’t every measurement of spin evidence for this? Fred Alexander Would you consider allowing one of your Patreon members to interview you for a Mindscape episode? dmi Doesn’t the Reeh-Schlieder theorem resolve the black hole information paradox? (edited) Rebecca Lashua On the eternal inflation wiki page it says, “the inflationary phase of the universe’s expansion lasts forever throughout most of the universe”. Does that mean that some finite distance from the earth there is a patch of spacetime undergoing exponentially fast expansion? Or are the parts of the universe undergoing inflation disconnected from our universe in some way? gillis15 Although the events were detected by LIGO and Virgo in 2020, a new paper just released confirms for the first time a black hole-neutron star merger. Would the black hole be able to tear apart the neutron star as the star spiraled in or would the neutron star cross the event horizon whole? naive bayesian How would a good-bayesian reconcile the following hypotheses: 1.) Reality is real 2.) Reality appears real because DeCarte’s Demon is tricking you 3.) Reality appears real because you’re a Boltzmann Brain 4.) Reality appears real because you’re a brain simulation 5.) Reality appears real because of literally any other ad-hoc tom-foolery example (cookie monster dream, last-thursdayism, etc) Given that all evidence shows up equally under each hypothesis, you’re left with your priors. Is there a reasonable way to come up with priors for hypotheses where the evidence doesn’t help you? Fabian Rosdalen Regarding vegetarianism; You’re the only person I’ve heard making a reasonable argument for justification of eating animals – namely their lack of ability to anticipate the future, their mortality and so on. The philosopher who got me convinced of veganism morally put forth the “name the trait” argumentation style of reasoning about these issues, namely that if this trait (or lack thereof) is sufficient for killing and eating another being, it should be so regardless of the species of that being. Example: “It’s ok to eat animals because they’re not as intelligent as humans” should then also conclude in “Therefore, it’s also ok to eat humans with equal intelligence to an animal”. Do you agree with this reasoning, if/if not, why? **************************************FTL Moshe Feder The two largest and most popular science fiction franchises involving space travel, Star Trek and Star Wars, share as part of their basic premise the common science fiction assumption that their methods of near-light-speed and faster-than-light travel somehow exempt them from relativistic time dilation. So that got me wondering about wormholes. If they could be made traversable and you used one to cross the galaxy and return, would the temporal effect of Relativity still apply even though you had never moved at relativistic speeds? Damon Winston Byrd I have to go to Alpha Centauri and don’t want to spend years getting there. I hop in my alcubierre drive and get there in only one year. After taking care of business there, I hop back into my faster than light alcubierre drive and am home again after another year of travel time. Did two years pass on Earth as I traveled? ******************************************************* Anonymous In episode #62 with Kate Jeffery, you both talked about how physicists are essentially certain that the universe won’t last forever, and about some of the processes that will cause the universe to wind down. You’ve also talked, in general, about how ironic it is when knowledge about physics stays stuck in ivory towers, when the explicit purpose of studying physics is to discover new knowledge for society. I’m curious about how the knowledge that the universe will definitely, permanently end has impacted society and culture…or whether that’s an example of something that the public really ought to know more about. LINEU D MIZIARA What would physics look like today if Feynman hadn’t invented his diagrams? Richard Kashdan Suppose I ask a priority question today. And then during the next month, I run the Universe Splitter app a bigly number of times to pretty much guarantee that I end up on a different timeline. Can I then submit another priority question next month? Louis B. How “sure” are we that things like e.g. hawking radiation are likely true? My understanding is that the prediction of Hawking radiation rests on both quantum mechanics and general relativity being useful approximations in the vicinity of a blackhole’s event horizon. But isn’t the entire problem those theories don’t play well together, in particular, in regimes precisely like a blackhole? Where, if anywhere, would you no longer “trust” these theories to be useful? [/accordion-item][/accordion]

[accordion clicktoclose=”true”][accordion-item tag=”p” state=closed title=”Click to Show Episode Transcript”]Click above to close. 0:00:00.0 Sean Carroll: Hello everybody, welcome to the July 2021 Ask Me Anything edition of the Mindscape Podcast, where we take questions from Patreon supporters of Mindscape. So if you’re not a Patreon supporter, that’s okay, you’re permitted to not be a Patreon supporter, but it’s way more fun and fulfilling in your life if you are. You can go to patreon.com/seanmcarroll, to sign up $1 per week or whatever it is, to support the podcast, and you get to ask AMA questions, and then I answer them and eventually everyone gets to listen. Sadly, not so sadly for me, but sadly for the AMA, there are so many people and so many good questions that I can’t answer all of them, so it’s Ask Me Anything, but I don’t necessarily answer everything. Sorry about that. 0:00:44.9 SC: I do try to get a representative sample, so if there are themes that keep appearing, I’m gonna try to at least answer someone’s question in that theme, and I think we’ve hit the point now where there are so many questions that I’m probably answering less than half, so really don’t feel bad if your question is not being answered, it’s not you, it’s me, as I said before, I try to pick out the questions where it’s not only a good question, but something where I in particular have something interesting to say about it. I will say, I like to give advice to people who wanna know how to get their questions answered. We say in the instructions, shorter questions are preferred. I know that some people seem to find that a difficult piece of advice to take seriously, but really, [chuckle] if you wanna get your question answered, shorter questions definitely do help you have that happen. 0:01:34.7 SC: It’s not enough, you can ask a short question, it doesn’t get answered, but it definitely helps me say, “Oh yes, I could answer this in a very brief period.” Also shout out to the Patreon supporters who suggested that we finally get a merch store here at Mindscape selling T-shirts and mugs and things like that, so we’ve set up one on TeePublic. You can find a link to it on The Mindscape webpage, which is preposterousuniverse.com/podcast. On the right-hand side, there’s a little link to T-shirts, mugs, and more. You get some designs there, maybe more will be coming in the future, who knows, but anyway, you want a sticker for your laptop that says Mindscape, or maybe you want your mask to have the core theory equation on it, there is the place to go. So with that, let’s go. [music] 0:02:37.5 SC: Eric Chen says, “What’s your take on Pascal’s wager? Do you think there is a simple explanation for where the argument fails, and if so, do you think you have a satisfactory account of how to make decisions, do ethics when infinities are at stake?” So for those of you who don’t know, to be honest, I’m not gonna get this exactly right, Pascal’s wager, but the basic idea is… Look, if God exists and God wants you to worship Him and promises you infinite time, eternity of pleasure in heaven, then it’s worth almost anything to get that if it’s a possible thing, even if you think that God isn’t very likely in your prior, the benefits of believing Him and worshipping Him, apparently are so great that we should just do it, even if there’s a 1% chance times infinity reward, then you should go for it. So I think that there’s two reasons. 0:03:28.2 SC: I’m probably butchering that by the way, ’cause I haven’t looked it up recently, but that’s the basic gist of the idea, and I think there’s two things going on: One is kind of trivial and one is a little bit more profound or difficult. The trivial thing is, you could say that kind of thing about all sorts of different propositions with very, very different outcomes. There’s a chance that God exists and would be really annoyed if you believed in Him, or worshipped Him for the wrong reasons, and those reasons might include Pascal’s wager and therefore, God would send you to hell for eternal torment if you believed Him for that reason. So that will be an argument to not do it. And all these arguments are a little bit silly because how do you actually decide what the entire set of things are. 0:04:15.7 SC: How do we decide which religion to be, if this is the kind of argument that we’re trying to follow? So just out of the gate, I think that Pascal’s wager is not a very good argument at all. There is, as Eric, you say in your question, a more interesting sub-question here, which is the infinity question, right? If you believe that more generally, there’s some kind of argument for maximising something, utility or reward or likelihood of success or whatever, and there’s something that is unlikely, but it’s then multiplied by infinity. How do you deal with that? It seems to sort of pin the needle to, you should definitely take this seriously even though a priori it’s very unlikely, and I think that that’s an argument for not taking these infinities seriously. 0:05:00.8 SC: I think this is not what you’re asking, but this is one of the reasons why heaven and hell are just kind of silly as concepts. It’s the human inability to distinguish between infinity and a very big number. We can imagine being happy or being sad, being rewarded or being tortured for a brief period of time, and for a relatively long period of time on human scales, so we just say, “What it we were infinite? What if that went for eternity?” We don’t sit back and ask whether that makes any sense, could you really even be happy for eternity? Is the idea of heaven a sensible concept to a human being? 0:05:41.0 SC: I doubt it personally, I don’t even think that that’s something that makes sense. And of course, a real religious believer would be, well, yes, your human capabilities here on Earth are not up to the task of comprehending the magnificence of heaven, and maybe that’s true, but then that means I can’t evaluate how seriously to take it here, if that’s the question I’m asking myself. I do think that in other contexts, these questions of dealing with infinity are subtle, and I’m not an expert on how to do it. In this context, I just think that that’s kind of a silly proposition you’re being forced to consider in Pascal’s wager. 0:06:18.3 SC: Daniel Westwater says, “Given that there’s more evidence for galaxies without dark matter, do you think it would be possible for the universe to exist without it? And what would it look like if so?” So there’s two things going on here. Also one is the straightforward answer to your question, which is, “Sure, the universe could exist without dark matter,” maybe you have something more specific in mind than the existence of the universe. The existence of the universe doesn’t care if there’s dark matter or not, you could have a universe that has another stuff in it or no stuff at all. As far as we know, at least, it might look different. Maybe what you mean is that the universe would look sufficiently different without dark matter that stars or galaxies wouldn’t form, and then maybe you wouldn’t have life, and there’d be some anthropic argument against that kind of universe, that’s possible. 0:07:04.0 SC: We can certainly imagine arguments like that, and people have made arguments like that, in fact, in the case of dark matter. It’s a little weak, because I think what would actually be the case if you didn’t have dark matter… Sorry, to back up a little bit. The idea is that dark matter has no restorative force when it becomes condensed into a region. So dark matter falls into a region under its gravitational pull, and there’s no he-door pressure pushing it back like there is with ordinary matter, whereas there’s also no stickiness or dissipation, so if you had nothing but dark matter in the universe, dark matter would sort of clump together a little bit, but not too much, because the dark matter particles just go right through each other when they begin to clump together. 0:07:56.8 SC: On the other hand, ordinary matter also clumps together a little bit, but not too much because it clumps together, but then it heats up and pushes apart. So, in some sense, it’s helpful to have both dark matter and ordinary matter working together. So in the real world, it might have helped the formation of early stars that we have dark matter in our universe, but that becomes a quantitative question, right? How much did it help? So you don’t need that many stars in the universe to have one available for life forming, right? So even if you might say, “Well, maybe there would only be 1/1,000th of the number of stars in the universe if they were in dark matter,” that’s still plenty to make life and things like that, as far as I know anyway, so I’m skeptical of those anthropic arguments also. 0:08:43.9 SC: But the real reason I wanted to answer this question, to address this question is because I do think there is an interesting point to be made here that people sometimes miss. When we say, as you said, there’s been news lately, if I’m getting it right, about finding galaxies that don’t have dark matter in them, and of course in certain circles, whether or not dark matter exists and the role it plays in cosmological model building is a little bit sticky. So what I wanted to emphasise is the counter-intuitive fact that finding evidence for galaxies without dark matter in them is evidence for dark matter. [chuckle] Now, why would that be true? Why would finding the absence of dark matter be evidence for dark matter? 0:09:28.3 SC: Well, the answer is the alternative to dark matter is something going on with gravity; some misunderstanding that we have and how gravity works on large scales when you get galaxy-sized things, but the thing about that hypothesis that something is going on with gravity is that it’s pretty universal. If you’re saying that the stuff of the universe is just the stuff we see and know about, the ordinary matter, the stars and gas and dust, and we don’t understand exactly how its gravitational field arises, but it arises from that stuff, then you have, no matter what your theory of gravity is, an immediate prediction that all clusters of similar kinds of stuff, similar amounts of stuff, will have the same gravitational field, ’cause that’s that visible stuff that is creating the gravitational field. 0:10:18.8 SC: Whereas if your theory is, well there is both ordinary matter and dark matter, then the dynamics of galaxies and sub-galaxies and clusters and things like that can be such that you have two equal sized collections of stars, one of which has dark matter associated with it, one of which doesn’t. So that dark matter hypothesis, even though usually the amount of dark matter, it tracks in some weak sense the amount of ordinary matter, it has that extra degree of freedom that lets you explain the existence of galaxies without dark matter and therefore just ordinary gravitational dynamics in a way that a theory that just modifies gravity wouldn’t let you do. So the evidence for galaxies without dark matter is evidence that dark matter is on the right track. 0:11:07.4 SC: Andre Dinu says, “In your latest podcast with John Preskill you touched upon the possibility that physical space and distances between objects are emergent phenomena arising from quantum entanglement. You say the following, ‘It’s not the entanglement between you and me that gives us the distance. It’s the entanglement between invisible degrees of freedom in the space between us, which is a little bit hard to visualise.’ Can you expand on this?” Well, I can try. I did write a book, [chuckle] Something Deeply Hidden, in which I talk about not only many worlds quantum mechanics, but also the emergence of spacetime. Part of the difficulty of this idea is that it is just a difficult idea, we’re asking you to grasp some purely abstract things. I’m not saying that spacetime is made of little pixels or atoms or anything tangible like that, it’s just made of abstract quantum mechanical degrees of freedom that have entanglement between them. 0:11:57.1 SC: It’s hard to visualise, it’s hard to wrap your brain around or to get your hands on it in some sense. So the idea is that there’s a quantum mechanical wave function that can be expressed as quantum mechanical degrees of freedom with different amounts of entanglement between them and the different amounts of entanglement knit together to form a more or less smooth three-dimensional spacetime. So the amount of entangled between these invisible spacetime degrees of freedom has a very specific geometric structure that fits nicely together to form a smooth manifold. Now, the huge question, and there were some other questions in the AMA that I cut out because they were little bit redundant with this one, so consider this an answer. 0:12:43.9 SC: The huge question is, even if the entanglment between these degrees of freedom can be like distances in space or spacetime, does that mean that they are distances in the space or spacetime? Well, good question, I don’t know. That is one of the things that we’re trying to understand. When you say there is a distance between objects in space or in spacetime, for that matter, you have some operational notion in mind about what that means. What you’re saying is that it takes a certain amount of time to get there, if you’re at a certain velocity or signals travel between this place and this place in a certain way, and so what you need is a dynamical theory of travelling. Distances don’t mean anything really until you can travel from one point to another. 0:13:29.3 SC: And so what you have to say is that the entanglement structure of these abstract degrees of freedom has an effect on the propagation speed of signals in this emergent spacetime. And there’s some hints that something like that might actually be true and we’re on the right track, but I would say we’re very far from completely spelling it out. So to add to the difficulty that all this is very abstract and hard to visualise, is the difficulty that we’re not there yet, it’s a very preliminary idea, we don’t have the full picture in mind, so I can’t really tell you much more than that. Sorry. 0:14:04.7 SC: Richard Riley says, “You told listeners a couple of months ago that you won’t be renewing your research professor contract at Caltech, and will focus instead on your podcast, Patreon, YouTube work, and your other writing plus your Santa Fe Institute activity. Are you concerned that not being fully in the mix of top research university will make you drift away from the centre of the physics cosmology world, no matter what else you may be doing?” So there’s two things to say about that: One is prior to… There’s a prior question here, you know, I think either I am not clear or sometimes people jump ahead of what I say to extrapolate beyond what I say. So it is not true that I am leaving Caltech to focus instead on podcast Patreon YouTube. That’s not true. 0:14:43.8 SC: What I meant to say, and may or may not have actually said this is, the fact that I am doing well with the podcast and with Patreon and things like that, gives me the freedom to contemplate doing something like that. I’m not at all devoted to doing it, I don’t know, my future options are quite up in the air, and in fact, the possibility that I do land at a university as a professor is pretty high. There’s plenty of universities, not plenty, there are universities who are in touch. Let’s put it that way, and would like to have me there and I need to think about how best to maximise the work I want to do. And so I don’t know what that will be yet. So to get into the actual… 0:15:29.5 SC: That’s the preamble, the actual question is a very good one, what is the benefit versus disadvantage of sticking around top research university? I think there are clear benefits and then there are also disadvantages, and so you have to weigh them. A clear benefit is that you’re surrounded by other really smart people doing interesting things, that’s the single biggest benefit. There’s sort of sub-benefits to that, you have graduate students or post-docs, you can teach and that can be inspiring, but the most important thing is that you’re in a network of people who can both give you ideas and also act as reality checks on the ideas that you have. 0:16:10.8 SC: I think that overall, if your goal was to be a productive scholar, intellectual, researcher, having that network of people to talk to is just super-duper crucially important one way or the other. So, being at a university is the most straightforward way to have that. There’s another way I can imagine having that by sort of bouncing around. Right? Part-time at a place like Santa Fe, visit other places. I’m spending this fall visiting Harvard, for example, so I could easily imagine being a gentleman researcher scholar, bouncing around from place to place, visiting places, mixing it up with people, and the freedom to do that would be provided by the fact that I don’t need to earn a living as a university professor, if that’s the direction I go to. 0:17:01.2 SC: Also there’s more down-to-earth benefits of having institutional affiliation. Having graduate students, being able to hire postdocs, apply for grants is a much easier thing to do and things like that, that part would… That really means that the Santa Fe part-time faculty position that I have, it would be very useful if that were the root that I decided to take, those sorts of things become much more possible. And at the same time… Then on the flip side, I should say, there are disadvantages to having a university position. It’s work. You gotta teach and you gotta be on committees, and you’re… My Caltech position in particular, which was a research professorship, since my research interests have shifted a lot and they don’t really overlap with the people I’m affiliated with at Caltech so much anymore, it became less of a good fit. 0:17:56.4 SC: Again, what I said before is I love Caltech, nothing against Caltech, it’s just not as good a fit for me as it once was. So, I think these are good concerns and it’s certainly easier now to imagine being completely independent ’cause you have the internet, you can listen in on talks, you can… The other great benefit that you used to have from being at a university is that you could go to seminars; seminars and colloquia or workshops that were nearby, people would come through and you could talk to them. Some of that is ameliorated by the fact that a lot of these things are online these days. So, there’s a kind of connection that doesn’t require that physical closeness. But still, as anyone who’s actually participated in this nose, whether it’s a workshop or conference or just your day-to-day job, a lot of the useful interactions with colleagues happen at lunch, or at coffee, or just sitting around talking, not necessarily in a formal context of asking questions at a seminar or something like that. 0:19:01.7 SC: So I’m a very big believer in the benefits of being at an institution like that, it’s just a complicated question, how to exactly balance things in a particular way, especially for someone like me. For a lot of people who just fit very, very naturally into a certain department, doing a certain kind of research, and that’s exactly where they’re happiest, it’s not much of a competition, that’s obviously the right thing to do. For someone like me who really values the interdisciplinary kinds of things, the ability to say, “Well, this month I’m doing physics, next month I’m doing philosophy, maybe someday I’ll do Economics or Biophysics or something like that,” the freedom is very, very helpful, so it’s an important set of considerations to keep in mind. 0:19:47.2 SC: Okay. Now I’m gonna group some questions together and I don’t think I’m gonna give a very profound answer to any of them, but there were a few of them. So, I think that boosts the chances of me answering the question when there are related questions and they’re about art. So Anita Tomasik says, “I wonder if there’s any tendency among theoretical physicists to prefer abstract art to figurative art.” Chris says, “What are your views on art? What is it? What is its purpose? How does it fit into the world view? Is there anything deep about it, or is it just a pleasurable activity, like eating a tasty meal?” And Alanna Salama says, “Who are some of your favourite artists out there, whether it was in the fine arts music, film literature, even comedy?” 0:20:30.8 SC: So I don’t think that I have, in any sense, a deeply thought out theory of art. I’m a big fan of art, and I take art in this case, not to just be painting or sculpture, or what are traditionally called The Fine Arts, but artistic things. So music, film, literature, even comedy as Alanna says, all of these counts, I think in what we would… I’m classifying as art for purposes of this discussion. I think that art is important. Chris says, “Is there anything deep about it, or is it just a pleasurable activity, like eating a tasty meal?” You know, I think that eating a tasty meal can be pretty deep. I think this is one of the pleasures of life. I think that that’s all you did, was eat tasty meals, if that’s the only pleasurable thing you did, that would end up being pretty shallow, just because it’s kind of one thing over and over again, and different kinds of artistic expressions do have different amounts of depth to them. 0:21:27.2 SC: Right? I love popular music, but a three-minute pop song can’t quite have the level of depth and complexity that an opera can. I don’t think that gives an opera any intrinsic objective superiority, but it gives it a different kind of experience and likewise tasty meals are good, but if that’s your only artistic experience, then you’re missing out on the literature and music and visual arts kinds of perspectives. So I think that there’s no… You shouldn’t be surprised if you know that I’m not a moral realist to learn that I’m not an aesthetic realist either. I don’t believe in the objective superiority of different works of art, I think that the important thing about art is its relationship to the experiencer of the art. Different kinds of people experience different works of art in different ways. And if one person, whether it’s me or someone else, sees or experiences a work of art and is deeply affected by it, whether it’s by getting pleasure from it or being provoked to contemplate things in a new way or just feeling some ineffable feeling of transcendence, then that is completely valid, even if someone else doesn’t feel the same thing at all. 0:22:52.3 SC: So I try very hard not to disparage works of art that I am individually not very much into, even if other people are. I think that discussions… There’s an interesting philosophy problem here because we tend… We judge things, and then when we judge things, we tend to attribute objective truth to our judgments, but in the case of art, I truly think we shouldn’t, even if you’re reviewing… If you’re a restaurant critic or a music critic or an art critic or whatever, in my mind, much more useful for someone like that to talk about the experience in a non-objectively judgmental way. If you like this, then you’ll like that, so many things… I made this comment on Twitter a few months ago, I guess, about book reviews, maybe it wasn’t book reviews in particular, but reviews of anything, book reviews, movie reviews, etcetera. 0:23:48.8 SC: There are so many reviews that say, this thing, this object, this action, this cultural product was a good example of X, but I wasn’t in the mood for X, I wanted Y, therefore one star. [chuckle] And this is enormously frustrating as someone who’s written books, and I can imagine it’s just as frustrating for someone who writes movies or produces movies. Right? “This movie is very depressing. I was in the mood for a comedy, I didn’t like it.” Just say it’s a comedy or just say it’s depressing, and maybe someone’s gonna be in the mood for that, and I think that would be the right kind of art criticism, one that really illuminated the different possible ways that a work of art could influence you and then let people react to them in their individual ways. Is there any tendency among theoretical physicists to prefer abstract art to figurative art? I don’t think so. 0:24:45.6 SC: I think that theoretical physicists have different attitudes towards art, some physicists are pretty down to earth about it, they want a picture of something on the wall. Others… Yeah, absolutely do prefer abstract art. So I think it’s an interesting question. I don’t have any data that would lead me to say, or at least I haven’t contemplated the question in a sufficiently systematic way to actually say whether there’s a preference one way or the other that is more tilted for theoretical physicist than for other people. Sorry about that. Who are some of my favourite artists out there? When it comes to the fine arts these days, I just can’t name names. Some of my best friends actually are fine artists here in LA, some of our very good friends are painters, and I love it, but I don’t have any knowledge of the field as a whole, it’s a whole huge world. Right? 0:25:41.2 SC: I do like abstract art, I like 20th century art very, very much, but I also like the older art, so going to an art museum is generally a positive experience for me, no matter what kind of specialty it is. I do like beauty in my art, I wanted to have some aesthetic sense. I think one of the failure modes of art usually, although not always, ’cause not everything is… You can’t make these predictions objectively, but for me, whenever a painter starts including words, text, into their art, it always makes it worse for me because the visual experience opens itself up into different kinds of thoughts in different ways that might impact one differently, depending on the time of day or whatever, the mood you’re in, whereas just putting a word there is just such a blunt instrument, as far as I am concerned. 0:26:39.4 SC: I love words in their own context, in the context of literature, but you can only put a few words onto a painting or whatever, and usually it acts to constrict and narrow the experience for me personally. So I started saying that because I’m trying to explain what kinds of artists I like when I go to museums or something like that. So look, if it’s Rembrandt or Michaelangelo, love ’em, they were amazing, Raphael, etcetera. Also, Picasso, Kandinsky, I have a Kandinsky T-shirt that I got when I was looking over things on a TeePublic when I was setting up my own T-shirt store. Brilliant Kandinsky T-shirt. I love the Abstract Expressionists, Mark Rothko, Jackson Pollock, Louise Bourgeois with these amazing sculptures, so I like lots of different things and I can’t even pinpoint, and many of these people are not alive ’cause I’m not a hip on the contemporary art scene. Sorry about that. 0:27:44.0 SC: Manuel Bevand says, “In a 2014 World Science Festival debate on quantum mechanics, you said something akin to, if a GRW turned out to be true, I would probably retire from physics.” You wrote JRW, but it’s GRW, Ghirardi, Rimini, and Weber. This is a theory of quantum mechanics, in which wave functions collapse spontaneously all by themselves, and so… Sorry, Manuel is asking, “Do you still think that today? If yes, could you elaborate on why a little more than you did at the time?” So to be clear, that was a joke. [chuckle] I would not necessarily retire from physics. In fact, it’s almost a private joke that I probably shouldn’t have said out loud because it’s an illusion to Joe Polchinski telling me years ago, like circa 1997 or ’96, that if they found the cosmological constant to be non-zero he would retire from physics because his reasoning was that the only good explanation for it is the anthropic principle… 0:28:42.3 SC: The anthropic principle is very ugly and dislikeable, so he would retire. It was just a joke when he was saying it. We did find the cosmological constant. He didn’t retire. And we joked about that. So the point is just to emphasise the fact that we physicists wanna find out the truth, we wanna find out how the world really is, and then you’re gonna work to understand whatever that truth happens to be, but we’re also human beings and we have preferences, we have things that ahead of time, we would consider to be more elegant solutions to certain problems. So I think that the idea that wave functions just sort of randomly collapse with a strong arrow of time, they collapse in one direction of time, but they don’t anti-collapse and it’s exactly tuned to be frequent enough to explain our experience of the macroscopic world. 0:29:32.6 SC: But infrequent enough not to show up in experiments that we’ve already done, that just seems to be asking a lot of… It just seems to be very awkward, okay? So I don’t like it, and I think that there is a much better explanation right there on the table, but I’m not gonna retire from physics even if it turns out to be true. I would suspect… This may or may not be true, but I would suspect that if it turns out to be true, there’s a more elegant, beautiful thing underlying that that will be remaining to be found. 0:30:01.2 SC: Nikolaj Sabo says, “I’ve recently come across an old Nautilus article that contrasted Neil deGrasse Tyson and Sam Harris as representatives of two very different but characteristic attitudes of public intellectuals. One is the very opinionated, the edgier, the better stands, and the other could be called the educator never taking sides or definite positions. My question is, what do you think about the role and responsibilities of public thinkers in our society? And do you really think there is such a spectrum, and if so, where would you ideally want to be on it, and where do you think you actually are on the spectrum?” So I wanna remove… This is an excellent question, it’s such a good question, I got to restrain what I wanna say about it, I could talk for a long time. 0:30:43.0 SC: But I wanna remove Neil deGrasse Tyson and Sam Harris from the question. Because as soon as you name individual names then there are fans or there are critics and they talk about those people, and I don’t care about that. I wanna talk about the underlying idea, because we all debate. There’s a long-going debate about how to be a good public intellectual and what that means, whether that every area says that the public intellectual is dying, etcetera, and the others will hold up their favourites. Okay. So it’s an important debate worth having. I do not think that this particular division into edgy and educating captures what it means to be a public intellectual. In fact, I would say that neither one of these categories really is what I think of as public intellectual. 0:31:32.3 SC: So let me explain what I mean by intellectual in this context. Intellectual doesn’t mean intelligent, you can be intelligent, but not an intellectual, or you can be an intellectual, not all that intelligent, really. An intellectual is someone who is characterised by, in my mind, and people can disagree, and it’s a word, who cares about the definition, but what I mean by it is someone who seeks truth as the highest value. They want to find out what things are true, how the world actually works, and this can be contrasted, the obvious contrast is with people who just kinda get along. 0:32:09.3 SC: They don’t work too hard to understand how the world as it is, but they have a pretty good view and they’re gonna get through the day. That’s most people in the world, but there’s another very important category that I don’t want to downgrade in importance, which is activists. An activist has a goal, and that goal might not be the same as the truth. The goal might be a very good goal; feed the hungry, cure diseases, fix the political system in some way. These are very good goals. And when you have these goals, if that’s your highest goal, that can get in the way of the goal of figuring out the truth and telling it. You would like to think, and it’s probably often true, that these goals are in alignment, but not always. If your view of how to make real change in the world is to lead a movement, whether it’s being a politician running for office or just being the head of some more informal coalition, you might come to the conclusion that talking certain ways, saying certain things, making certain activities and so forth is more important than just telling the truth or that there are some things that are true that you don’t want to make public. 0:33:29.0 SC: ‘Cause they get in the way of your goal. So I’m not disparaging that, activists are important in the world, but it’s not me and it’s not an intellectual. The intellectual is going to say, “Look, I’m gonna tell you what the truth is, I’m gonna trying to figure out the truth, and no matter what the implications are for what happens in the world,” which is why informally, I have a policy here on Mindscape that I don’t invite active politicians onto the show. Someone who was a politician and is now retired, maybe, okay. But once you’re a politician, your goals are a little bit different than discovering the truth, and I will stand up for the importance of politics all the time, and I’m pro-politics, but it’s a different thing than being intellectual. So intellectuals are ones who care about the truth, and even maybe you could add to that people who sort of have a process of finding out the truth, not just they care about it passively, but they care about it actively, they’re taking steps to understand the world better. 0:34:35.3 SC: Okay? And a public intellectual is one who does that in public, who does that in full view of the world. Most intellectuals are not public. And again, that’s fine. I don’t want most intellectuals to be public, frankly. Some of them are sort of a turn-off in the ways that they would talk about things, and there’s no necessary connection between finding the truth and talking about the truth out loud in front of an audience. I think both are important, but I think they’re different. So I think there’s room for people who are educators, but that’s different than being a public intellectual. An educator is someone who says, “Okay, here are some things we’ve figured out, I’m gonna explain them to you, I’m gonna teach you, I’m going to be pedagogical about it, this is an incredibly important thing to do.” 0:35:24.7 SC: In some way, this is journalism, all of journalism is, “Okay, there are some true things, I’m gonna explain them to you.” But to me, that’s not quite being an intellectual, because an intellectual is saying, “We don’t know what the truth is yet, we’re trying to find it. So we’re working in this process to learn about the truth, to consider different hypotheses, to make arguments.” The real difference to me between an intellectual and an educator is that an educator is basically hanging on to things we all more or less agree are right, at least the people who are more or less in the mainstream agree these things are right. Human beings are causing global warming, the Earth is round, we evolved from other species, these kinds of things not everyone sadly agrees are right, but there is pretty much a consensus. 0:36:15.2 SC: Whereas an intellectual is working at, “We’re not sure what is the right interpretation of quantum mechanics.” I can’t pretend just to be a disinterested educator where I talk about the right interpretation of quantum mechanics. I need to say on the one hand, “We don’t know yet, there are people who disagree and acknowledge that and put that front and centre.” And on the other hand, I wanna say, “But here’s why I think this one is true.” So an educator would say, “Here are what people say about the interpretation of quantum mechanics,” and intellectual would add, “And here’s why I think this one is true.” Which means that intellectuals can be wrong sometimes, but it’s a slight, but I think crucially important difference in your question or quoting from this Nautilus article, I guess, you’re glossing the idea of being an educator as saying never taking sides or definite positions. 0:37:11.0 SC: I think it’s a slightly unfair way of saying it. I mean, educators are saying, “Look, there’s a lot of stuff we know about the world that’s interesting and important, and we’re gonna try to share it.” That’s a very, very valuable important thing to do, it’s just a slightly different thing than the intellectual engaged in an argument. Intellectuals are out there on the boundary between what we know and what we don’t know, trying to figure out what is going on. And the thing is that this activity of being intellectual, being devoted to the truth and working out what the truth is, making arguments, listening to other people’s arguments, responding to them and so forth, there’s a tension there with the idea of being public. It’s not completely opposed to it, but it’s not completely aligned to it either, because let’s face it, you’re not that much of a public intellectual if no one’s listening to you. 0:38:07.6 SC: You can call yourself a public intellectual because you’re out there talking, but there’s a difference between talking in a town square with a crowd gathered around you versus talking the same talk in the middle of an empty field where no one’s listening. So to be a public intellectual implies that you want people to listen to you and you have some success in getting them to listen to you, and there you get into the question of the strategies or the issue of the strategies for engaging the public and getting them interested and gathering a big audience might also not be aligned with the strategy of telling the truth. And this is where the edginess comes in, the idea not just of edginess, but of contrarianism, of being a heretic, being radical. 0:38:53.5 SC: To me, all of these ideas, being a contrarian, being edgy, being radical, these are anti-intellectual ideas at a very fundamental level, not because you should never be contrarian or be a heretic, or be radical, or be edgy, but to valorise those things is not to valorise finding the truth. Sometimes the truth is conventional, sometimes the truth is the middle of the road, you have to be open to saying, “Well, maybe people are right,” and when people are right, explain why they’re right. I was invited to write a short piece for one of these edited volumes that John Brockman used to put together. John Brockman is the purveyor of the Edge website, and every year he would have a question centre that he would make into a book. 0:39:42.6 SC: So the question was, one year, what have you changed your mind about? And I explained in my little short essay, like two paragraphs, literally, when I was younger, let’s say in college, I did valorise being a heretic, going against the system, fighting the man, and so my little changing mind essay was being a heretic is hard work. [chuckle] Because it’s not good to just be a heretic for the sake of being a heretic, you have to be a heretic because there is some individual case where you think it’s worth going against the conventional wisdom, and so I did change my mind about that. And I’m still willing to go against the conventional wisdom, but I’m much more reluctant to do it because I understand that there is a sort of romance to it, an attraction to it for that reason, both individually and for the purposes of gathering an audience. 0:40:38.6 SC: That is dangerous to the value of being intellectual, which is what I actually value. If your first goal is being correct, that’s different than having your first goal being edgy. I remember when, somewhere on the internet, someone mentioned that they had started listening to my podcast, but they stopped listening and went to other podcasts because they were edgier. And with all due respect, my immediate response was, “Good riddance. You’re not the audience I’m looking for if what you’re looking for is edginess.” My audience, I’m hoping, is in it for the truth, and we don’t always know what the truth is, which is why I will on the podcast try to have people I disagree with, but it’s all part of the journey to find the truth, and sometimes the truth will be in the minority, the radical view, sometimes it will be in the majority and just go along and getting that balance right is difficult. 0:41:32.1 SC: And anyway, I think that public intellectuals are crucially important because I think that it’s more than just educating, it’s exhibiting the process of trying to figure out… The truth in the case of questions, it can be very, very hard. And that’s messy, you’ll be wrong sometimes, you’ll talk to people who you disagree with, but I wanna talk to people who I can engage with, and I wanna sort of set an example for constructive engagement between people who disagree in different ways. And some people are not gonna like that, it’s not gonna be some people’s thing, and furthermore, I have my own idiosyncratic things that I care about. I’m willing to be a bit more technical in the weeds, etcetera, than many other people who… In science communication anyway. 0:42:27.1 SC: And so that limits the audience that I’m gonna get in some ways, but that’s okay, ’cause it’s still a pretty sizeable audience. There are people I can reach with things like the biggest ideas in the universe, or many of the episodes of this podcast that I think want a little bit more than they usually get in the discourse and they’re not in it for the cheap thrills, they really wanna hear the truth, and the truth is thrilling enough in my view. So, I can carve out a niche, I’m never gonna be the super duper famous public intellectual science communicator or whatever, but I think my people appreciate what I am trying to do, and that’s what is very rewarding for me. 0:43:08.3 SC: Okay. Sorry if that was too long, but you pressed a button there. John Perry says, “Taking on board strands of various podcasts, what chances do you give to democracy winning out as the optimum model for national government? Given the increasing economic dominance and military power of countries like China and the level of digital monitoring of individuals, etcetera, they rapidly being able to achieve.” This is asking me in some sense to predict the future, what chances would I give to democracy winning out and I’m very bad at predicting the future. That’s one of the things I will acknowledge is not my area, not my expertise. And furthermore, I think it’s probably misguided to imagine there is such an idea as winning out when it comes to being the optimum model for national government. I think that what we will see here I just said, I’m bad at making predictions and I’m about to make a prediction, but it’s kind of the milk toasty prediction. 0:44:07.7 SC: My prediction is that we’ll continue to see many different models. That the equilibrium around the world is that there will be some democracies, some authoritarian things, some socialist things, some more radical, different experiments and different kinds of government and so on. What we can hope is to change the balance between these different kinds of things. I do think that China, in some ways, in many ways, is that a very natural world leading country, it’s a huge country, technologically advanced, speaks a common language, and they have a system of government that is very well adapted to rapidly doing things; its central planning, its ability to make choices and have them implemented very, very quickly. But of course that same organisation has huge disadvantages as well. You know, we talked with Henry Farrell a little bit about the fact that simply when it comes to solving social problems, there’s a case to be made that democracy is a better solution-finder than authoritarian systems are. 0:45:09.9 SC: But also there are a set of values to go together that both go into progress, whether it’s technological or scientific progress, or social or moral progress, and democracy. And Henry and I didn’t talk about this, but this is more… If democracy is a good problem-solving measure, then that’s a benefit, that’s a bonus, that’s a spin-off as far as I’m concerned. What matters to me about democracy is that it puts power in the hand of individual people, and that power helps unleash their creativity. Even though science and politics are two very different things, the underlying values of allowing people to try different things, to be creative, to go against the grain sometimes, are very, very important, both for democratic politics and for experimental theoretical sciences. So there’s a huge disadvantage that any authoritarian society puts themselves at in a world where scientific and technological progress are important. China has a big economy, they’re very, very good at making T-shirts and things like that. 0:46:26.0 SC: They’re not as good as we are, here in the United States or in Europe at scientific progress, at least not right now. Maybe they will be, but I think that part of it is the spirit of letting people be rebellious a little bit, goes hand-in-hand with the spirit of scientific advancement. Anyone who thinks that there is some lockstep scientific establishment that is working behind the scenes to squelch their ideas has never been to a Physics seminar and listened to people asking questions, people are not in lockstep. People like to disagree. That’s important. That’s part of the creativity. So all I’m adding up to is, I don’t know. I don’t know what’s gonna win out, there’s a lot that goes into these things and a lot of them are random, a lot of them are unpredictable, so I like democracy, I hope it wins more places than it loses, but I can’t be too sanguine about those things. 0:47:19.9 SC: John T says, “Many podcasts often include scientists as guests to speak on controversial topics, but the term scientist is vague, anyone can claim to be a scientist. So as a layperson listening to a podcast with scientist guests, how can I distinguish their credibility to speak with genuine authority, what heuristics can we use to weigh these opinions and protect ourselves against the fallacy of appeals to authority.” 0:47:43.6 SC: Well, appealing to authority is a fallacy, but it is not useless; it is complicated, and I think this is the most important first step in answering this question, which is a very good question, is that it’s complicated. So, you used the word heuristics, and I think it is the right one to use, rather than clear-cut algorithm, when you have people with different expertises and different credentials saying things that you yourself are not an expert in, who to believe, who to trust, whose opinions to weight in different ways is a very hard question. And I think that it is completely valid to weight people, not 100% or not 0%; to give medium-sized weights to a lot of different things, but there are things that matter more. 0:48:37.1 SC: So I think I’ve said things like this before, Julia Galef, when she was on the podcast, she and I talked about this. If someone is a world-renowned expert in precisely the thing they’re talking about, count them more than someone who is not. And that’s kind of obvious. It’s not really very helpful. But what does it mean to be a world-renowned expert? It means more than just, you got a degree. Look, I went to grad school. I have a PhD. I know other people who got PhDs, not all of them are super sharp, to be honest, some of them are brilliant, some of them are not, but it’s a start. Someone with a PhD in economics, you should count more in their opinions about Economics than someone without any PhDs at all or any special expertise in economics. That’s pretty obvious to do, but someone can claim to be an expert and maybe even have a PhD, but if you wanna get more into the weeds: What have they accomplished? What have they done? Have they written papers? Have they changed the field by having some good ideas that later turned out to be valid? What is their track record? This is the sort of second order questions you can be asking yourself. 0:49:52.0 SC: I think you correctly, maybe allude to the idea that if you have a PhD in economics and are propounding on politics or biology or the environment, then who cares about your PhD in economics, right? That’s not what’s really going to give you the expertise in this field, so you have to care about where their expertise comes from or where their credentials actually come from. And finally, people can be absolutely insightful and correct about ideas where they are not credentialed experts. So don’t discount people entirely just because they’re not experts in the field, but weigh their words carefully, that’s why this is a hard thing to do. Again, what is their track record in other areas? If you’re not a PhD in Physics, but you’re propounding on physics, what’s your track record? What have you gotten right in the past? I’m completely willing to believe that there are people out there who don’t have PhDs in physics, yet have really interesting things to say about it, but I also know for a fact there are people with and without PhDs in physics who say lots of spout, lots of nonsense about it. 0:51:02.1 SC: So it’s just a matter of keeping your bearings and not taking anyone too seriously if you don’t know what their background and track record is, and even if their track record is great, they could be messing up this time. And I know that’s a wishy-washy answer, but that’s the best you can do, if there’s a bunch of people… I guess, sorry… There is one other thing which is crucially important, which I take for granted sometimes. 0:51:30.4 SC: When people do proclaim things, okay, when they say, I think X is true, regardless of their credentials or their track record, the way they say it, and the arguments they give in favour of it, can also be used as a measure of how seriously to take them. In particular, the short version of this is, what is the evidence that they’re giving you, that they appreciate why they might be wrong, that they understand the counter arguments to it, right. If they are just straw manning the counter-arguments, or if they’re just sort of snarky and sarcastic, or if they show no sign that they even know what the counter arguments are, then you should discount what they’re saying compared to people who go, “I know what you’re thinking, you’re gonna think it’s not true because of this, but here’s why it’s true, anyway.” 0:52:24.5 SC: And this kind of technique is just as true for research papers in the technical literature, in the professional literature as it is for people expounding on podcasts and things like that. When people know why they might be wrong, it’s much more worth listening to what they’re trying to say because you can think that those people have thought through what the different arguments are, rather than just falling in love with one point of view and stating it. 0:52:54.0 SC: Okay, Alexander Cordova says, “When discussing quantum mechanics, the example, that is always used is the electron, however, it’s also stated that everything is quantum in nature at a small enough scale, do protons, neutrons have wave functions as well?” 0:53:07.0 SC: Sure. Everything has wave functions. That’s the easy part. Let me just explain a little bit about why the electron is always used as the example. It’s sort of a combination of two things: One is, the electron is, as far as we know, elementary; it is not made of other particles. So, protons and neutrons are made of quarks and gluons. So that just makes things a little bit fuzzier, when you talk about protons and neutrons. It’s actually perfectly okay to, in some circumstances, treat protons and neutrons as fundamental particles. In other circumstances, it’s not, right? 0:53:42.5 SC: You have to take into account that internal structure, so it becomes more complicated, whereas electrons, as far as we know, just are fundamental particles, so that complication doesn’t get in the way. The other is, electrons are the lightest charged particles in nature. They are the lightest particles that have electric charge, electrons and positrons, which are the anti-particles. So what that means is that since electromagnetism is really the force of nature that makes everyday world go, electrons are doing most of the work, right? Electrons are easy to manipulate, electricity is mostly the motion of electrons, ’cause electrons are lighter than protons and neutrons. In principle, electrons going left is equivalent to protons going right. But it requires a lot more energy to get protons going right than electrons going left. So in practice, when you have electricity going down a wire, the electrons are doing almost all the work. So that’s why they’re used as the examples in quantum mechanics; it’s easy to manipulate electrons compared to protons or neutrons, but everything has a wave function. Don’t worry about that. 0:54:48.0 SC: Paul Cousan or Cousan says, “Thibaut Giraud, a French YouTuber recently did an experiment on his philosophy channel Monsieur Phi or phi, I don’t know how French people pronounce the Greek letter phi. I don’t know how Greeks pronouns the Greek letter phi for that matter. He surveyed 12,000 of his viewers with carefully corrected thought experiments about free will and morality, as an example of an interesting and somewhat surprising result, around 80% were intuitive compatibilists, what do you think about the idea of experimental philosophy?” 0:55:20.8 SC: I don’t think it’s philosophy. I think it’s an interesting thing to do, but it’s more like you’re learning about psychology, you’re not learning about philosophy. I like to think that ideas in philosophy, whether it’s free will compatibilism or anything else, stand and fall on the reasons for them, not on a poll, not on a popularity contest. I do think that anti-free will people sometimes fall into the trap of saying, well, this or that… Maybe pro-compatibilist free will people too, I don’t know. 0:55:56.0 SC: But anyway, people sometimes fall into the trap of saying, that’s not what the people really have in mind when you say free will. Okay, I don’t know what the people have in mind, that’s not an important consideration for me. I’m just trying to get the most useful vocabulary to discuss the world. I wanna be right, and I wanna sort of figure out how best to characterise the fact that human beings make decisions, okay? I find talking about free will for that purpose is perfectly valid. If you don’t, that’s fine, and I honestly couldn’t care less what the majority of people have in mind as long as we can be clear. In fact, my strategy, it has completely failed so far, but the strategy I’m gonna keep trying to plug away at is, whenever someone really wants to have a discussion about free will, I’m gonna say, I tried this with Sam Harris, it didn’t work with him either, I’m gonna say, I’m happy to do that as long as we never use the phrase free will, because as soon as you use the phrase free will, you have this incredibly long, tedious, boring and completely pointless conversation about, what do you mean by free will? 0:57:03.1 SC: So just don’t use that phrase, if you can’t agree on what it means, just don’t use it; use what words would actually convey the meaning that you have, and I’m perfectly happy to do that. 0:57:11.9 SC: Alright, Connor Scott says, “I know the Schrödinger equation doesn’t predict wave function collapse, but it definitely predicts wave function branching, sorry, does it definitively predict wave function branching in the case where there is no collapse, is that something everyone can agree on?” 0:57:28.1 SC: There are so few things that everyone can agree on in this game. They can agree on what the Schrödinger equation predicts, in terms of given some wave function and given some Hamiltonian, what does the Schrödinger equation say that wave function evolves into? But you use the word branching in your question, so for example, if you have an electron that’s in a superposition of spin up and spin down, and you run it through Stern-Gerlach experiment where there’s a detector and an environment and so forth, everyone agrees, that the wave function of the universe, if all it does is, obey the Schrödinger equation would evolve into a part that says the electron will spin up, it was measured to be spin up and the environment noticed that, plus a part that said the electron will spin down the detector measured it to be spin down, and the environment noticed that. 0:58:26.4 SC: I can’t promise you that everyone is going to be happy to call those two parts of the wave function, branches. There’s an argument that goes into saying there’s a reason why we should call this branches, it has to do with decoherence and the independence of the future evolution of these two parts of the wave function, the fact they won’t interfere with each other, etcetera. But everyone agrees that the Schrödinger equation predicts that that’s what a wave function should looks like. That’s why people who believe the Copenhagen interpretation or something like that, don’t accept the Schrödinger equation to describe quantum measurements ’cause they don’t believe in those two parts of the wave functions still surviving that measurement event. 0:59:09.0 SC: Julian Reif or Reif says, “Carlo Rovelli wrote the following about many worlds interpretation in his new book, Helgoland, ‘It is not enough to know the wave function in Schrödinger’s equation in order to define and use quantum theory. We need to specify an algebra of observables, otherwise we cannot calculate anything and there’s no relation with the phenomena of our experience. The role of this algebra of observables, which is extremely clear in other interpretations is not at all clear in the many worlds interpretation.’ What do you think about this criticism?” 0:59:39.1 SC: Yeah. I think it’s not a criticism, I think it’s just a statement, as a many worlds proponent. So I would agree with the claim, the algebra of observables, the role of the algebra of observables is extremely clear in other interpretations not, and it’s less clear in many worlds, why? Because in all of the other interpretations, you put the algebra in there by hand, you add extra super structure, whereas many worlds is the simplest, cleanest interpretation of quantum mechanics. So when it comes to things like observables, just like things like branching and probabilities and so forth, you have to derive them, they’re emergent. You don’t just put them in by hand. So it’s less clear in many worlds because it’s more work, and this is something where I think any fair observer would say, “This kind of feature of many worlds is both an advantage and a disadvantage of the theory; it’s an advantage because rather than just positing something, people who believe in many worlds are saying we can derive that thing. 1:00:49.4 SC: It’s a disadvantage because maybe you can’t derive that thing, maybe you think you can, and you’re just fooling yourself and it’s more work to show it, right?” So I take the worry very, very seriously, the paper I wrote just last year with Ashmeet Singh on quantum mereology tackles exactly this problem of showing how the algebra of observables arises in the many worlds interpretation of quantum mechanics. I think we have a pretty good explanation of why it does, so I’m not that worried, but I want to agree with the problem, and then I wanna say, “Yeah, but we can solve it and solving it is good, right? That’s what we’re here for.” So by all means, criticise away, especially those criticisms that I have answers to. That sounds good. 1:01:38.3 SC: Richard McLay says, “I recently listened to a somewhat frustrating podcast on post-modernism and post-truth, what are your thoughts on the existence of objective truth?” 1:01:47.5 SC: So I don’t know why your, podcast you listen to is frustrating, so I’m not sure what side it was frustrating on. Look, I believe in objective truth inn the following sense. I believe in objective reality. I believe that there is a world that objectively exists out there, that we’re part of, okay? And that there are certain statements we can imagine making about the world which will be correct, and certain ones that will be incorrect about the physical behaviour of that world. On the other hand, I’m more pro-post-modernism than most other scientists or physicists in particular, and part of it is that physicists are not that patient with a whole bunch of things. Physicists… I think this is gonna come back later in the AMA also maybe, but we talked already about intellectuals and this idea of being intellectual, being devoted to truth. It is related to, but not the same as being broad-ranging in the kinds of truth you care about. 1:02:53.2 SC: So you can be very, very devoted to finding the kind of truth in your particular field and/or your subject of interest and just less interested in other fields, that’s a perfectly valid thing to do. Again, no one forces you to be wide-ranging in those ways, and in fact, a lot of the… Yeah, this will come back… I should have grouped these two questions together, we will come back to this later in the AMA, but a lot of the attitudes of physicists towards other fields are because they’re different than physics. They go by different standards, they use different techniques and so forth. And post-modernism, in particular, pushes physicists’ buttons, it pushes scientists’ buttons in all sorts of bad ways, and also just to layer on more throat clearing here. 1:03:44.5 SC: There’s a lot of bad post-modernism out there. It’s not hard to find people just saying very, very silly things in the name of post-modernism, but I’m not interested in the bad post-modernism. I wanna ask, is there any good post-modernism? Is that any useful? And I think that there is, there’s a point, there’s an insight, an essential insight in the postmodern project, which is that even if the world exists and is objective, we don’t know it clearly and objectively, and in an unmediated way. We are tiny, finite, biased beings, and so even though we exist in the world, the stories that we tell about the world, whether they’re literally stories or whether they’re highly developed and highly rigorous physics theories are partial attempts to capture a bit of the world, and we should be very… They’re very fallibilistic, we should be willing to change them when new data comes in. And when it comes to physics, that insight is kind of trivial and not very helpful, right, sure. Well, if new experiments come in that tell us to change our theories, we should do that, right. 1:04:55.3 SC: But we’re pretty sure that apples will still fall from trees, no matter what we learn about gravity, a 100 years in the future, or a million years in the future. When you turn it to things that postmodernists more typically care about, which is the social world and supposed truths of ethics or morality, or society or politics, or literature or art, as we were talking about art before, then the fact that these truths are fallible and partial is much more important to keep in mind because you can be fooled much more easily when you talk about these harder areas then when you talk about Physics. Physics is easy, it’s easy to do experiments to find out whether you’re right or wrong. It is easier in other areas, to fall into a trap that you think you got things figured out, that you have a way of talking about the world that is just correct, even though you’re actually embedded in a web of presuppositions that should be questioned, right? And I think that’s the usefulness of post-modernism, that postmodernism gives you a toolkit for questioning these presuppositions, for saying that, “Well, you know, I know that sounds like some objective truth about the world, but have you thought of this?” 1:06:12.5 SC: That’s the good variety of post-modernism. Post-truth, I’m not even sure what that means. I think that the phrase post-truth is almost always used as a slander against people you don’t like. I don’t see anyone really who is positively in favour of post-truth. I don’t see anyone making the case that truth is overrated and we should get rid of it, not any sensible people, again, there’s always crazy people out there. But the idea that truth is this idea that has outlived its usefulness is really not one that is gaining a lot of currency among smart people in the world today. Usually, the accusation of being post-truth is made against your enemies to say that they are distorting the truth in some way or another. And again, that absolutely happens, especially when it comes to politics and activism. When your first loyalty is to a political goal rather than an intellectual goal, then you will often end up distorting the truth and everything I just said, applies equally well, no matter which side of the political spectrum you are on. 1:07:23.3 SC: But I think it’s a bit lazy and useless, that particular accusation, this post-truth accusation, because it misreads what people are trying to do because I don’t think that people describe themselves as post-truth, usually. How they would describe themselves, which is always what I think we should be asking, “What is the people who really believe this, what is their self-understanding?” They might have different ways of finding the truth and judging the truth than you do. They might count different pieces of evidence and weigh them differently than you do, and that doesn’t mean that they’re not wrong, in a real objective sense, but if they’re wrong, confront them on why they’re wrong, be honest about it. What is the evidence? Where are they going wrong? Where are they making the mistake? To just sort of say, “Well, you’re not even aiming for the truths,” I think it is just a kind of lazy criticism that is rhetorically beneficial to your side, but again, it’s that activist mindset rather than an intellectual mindset in my mind, I think… And all of that, everything I just said is usually, but of course, there’s plenty of exceptions out there. 1:08:31.9 SC: Casey Mahon says, “Given that all branches of the wave function are equally real, there would be presumably some bizarre, unexplainable things happening in some branches; a car passing through a wall, an elephant appearing out of thin air, etcetera. Given the enormous number of times the wave function branches every second, is it possible that some of these freaky events have occurred in the history of our own branch, thus accounting for some ghost stories or other bizarre phenomena?” 1:08:57.0 SC: So, as I said before, whenever a question starts with, “Is it possible?” The answer is usually gonna be yes, it’s possible. But look, you have to just ask this question a little more carefully, you have to think it through. Yes, in the branches of the wave function in Everettian quantum mechanics, some unlikely things will happen and many things happen in the world, and you wanna say, “Okay, there’s a number, less than one, the probability of some unlikely event, and then there’s a number of times that we look at events.” Okay, so you have some big number, the number of times you look at events, times, some small number, the likelihood of each unlikely event, and you wanna say, therefore, when you multiply these two numbers together, is it likely that unlikely events happened, Okay. 1:09:43.8 SC: But you haven’t quantified what either one of those are here, and roughly the answer is that, the unlikely events are really, really unlikely, so unlikely that even if a lot of events happen, you’re not gonna see that many very, very unlikely events. So in quantum mechanics, there are events like I put the coffee cup on the table, and it quantum tunnels right through, what is the likelihood of this happening? And you can say like, “Look, there’s been a lot of coffee cups put on tables throughout human history, surely it should have happened.” Well, no, if the probability, if the number of coffee cups that have been put on tables is, I don’t know, 10 to the 20, that’s way bigger than the actual number, but okay, that’s a lot of coffee cups being put on tables, but the chances that the coffee cup tunnels through the table is 10 to the minus a million, which is a very, which actually overestimates the probability for it happening. 1:10:36.2 SC: So what this means is that no, you can’t just go qualitatively from some unlikely things are gonna happen to probably they explain things. It’s way easier to explain ghost stories by saying people mistook what they saw late at night in the dark, that’s a way easier explanation than invoking the many worlds interpretation of quantum mechanics. 1:10:56.9 SC: Okay, David Frank says, “If you were Daryl Morey, former Mindscape guest and president of operations for the Philadelphia 76ers, what would you do with Ben Simmons this off season?” 1:11:08.5 SC: I know not everyone here is a basketball fan, but I thought this is an interesting enough question to address because the reason why I thought it was fun to have Daryl on the podcast is because deciding what to do as a general manager in a basketball situation, is it completely analogous to a whole bunch of other very real world decision problems that we make. The difference is that in basketball or in other professional sports, just like in poker, which we’ve also talked about, the results are very quantitative and very quick, right? So, you can be a little bit more careful about saying, “Well, should I make this decision or should I make that decision?” In the real world when it’s like, “Should I get married or should I not get married?” It’s hard to know whether you made the right decision or not, ’cause it’s hard to see what would have happened in the other choice. So Ben Simmons is the point guard all-star basketball player, young player still, 24 years old for the Philadelphia 76ers. 1:12:05.6 SC: He did not perform well in the playoffs this year, I think that is safe to say. He is a very polarising figure among NBA fans. Because the basic thing in basketball all you need to know is you put the ball in the basket to score points, and the team that scores the most points wins. So, that sounds like there’s only one thing that happens, is you put the ball in the basket, but the other thing that happens is that you stop the other team from putting the ball in the basket, but it’s way easier to be quantitative and judgmental about people’s ability to score, to shoot the basketball, to put the ball in the basket than it is other things, playing defense, passing, rebounding and so forth. 1:12:48.4 SC: And the reason why Ben Simmons is so polarising, is ’cause he’s clearly super-duper talented, but he’s absolutely super world class at everything, but putting the ball in the basket; the one skill set, especially for the point guard, the guy who runs the offense, the one skill set that is really, really important and very easy to see when it’s on display is shooting the ball, and Ben Simmons is not any good at that. And some of it is, who knows why? Some of it is psychological, some of it is just that he wants to do other things, and he is actually quite smart about it. Some of it, people claim that he shoots with his left hand even though he is naturally right-handed, which is true, but can you overcome that? So there’s a lot going on. Simmons is very good at passing, very good at running, very good at defending, he’s arguably the best defender in the NBA, how do you value a player like that? So that’s a good question. Yeah, I really don’t know. I think that, the reason why sports fans are sometimes very annoying to read online or even to talk to is that they ask questions like this, “What would you do with Ben Simmons this off-season?” 1:14:00.0 SC: Knowing perfectly well, the options are, you keep him on the team, but you try to smooth over those rough edges, get him to shoot more, get him better at shooting, etcetera, or you trade him. But to trade him requires another team to offer you something in return, and in fact, Ben Simmons is very valued around the league; people will offer you something in return, but they know, everyone else knows that he didn’t do that great in the playoffs. 1:14:26.7 SC: That the fans are sort of grumbling about his playoff performance, etcetera. So maybe the other general managers are not gonna offer fair value, you just don’t know. As a basketball fan, I’m a fan, I’m not the general manager, I don’t have a say in what happens. So I tend to root for my players, right? It might be true. Look, the short version is, if you could get a trade that you could get Damian Lillard on the 6ers in favour of Ben Simmons, I would do it, I would sort of do it right away. I don’t know if that’s a possible thing to do. If we don’t trade Ben Simmons and keep him on the team, then I would actually be perfectly happy with that. He’s a really, really good player, I don’t wanna trade him for worse players, just for the sake of saying that we did something. So we’ll see what happens. I’m glad I’m not Daryl Morey in this particular case, but it’ll be an interesting off season, 6ers fans. 1:15:26.0 SC: I’m gonna group a couple of questions together here. They’re all about expanding space, so one is from nuzzly fuzzly torstein pill of the satanic temple. I think some of you people are editing your names just to get me to say them out loud in these AMAs. The question is, “It’s my understanding that the universe has a kind of resolution of planck size, as the universe expands, is the planck size ever so slightly increasing too or is the universe increasing in the amount of, to put it in cutesy way, planckies?” William Garrett says, “Since matter including ourselves is mostly empty space and the universe seems to be expanding, are we expanding with it? Is subatomic space expanding similarly to outer space?” And then Paul Hardy says, “I read an article suggesting that inflation theory might mean that space has been expanding forever. I’ve also heard that space might not be fundamental, are these two ideas contradictory?” 1:16:16.1 SC: So there’s a bunch of ideas that are similar in these three questions, even though they’re different. One thing that I do want to emphasise is that, don’t believe that the universe has a resolution of the planck scale; that is not the right way to think about the planck scale. So, the planck scale is a measure of length, there’s also the planck time, the planck energy, planck mass, etcetera, that you get from combining Newton’s constant of gravity, planck’s constant of quantum mechanics, and the speed of light. I think that the correct way to interpret that is, if you have processes in physics that in some sense are happening at the planck scale, so in other words, things that are squeezed down into lengths smaller than the planck length or processes that happen faster than the planck time or collisions that exchange more than the planck energy, in any one of these cases examples, then you will have to take into account gravity, relativity, and quantum mechanics, because all of those constants are in that formula for these different length scales. What does it mean to take into account gravity, relativity, and quantum mechanics? Nobody knows really. Okay. 1:17:34.3 SC: It does not mean that there’s a little pixels of planck volume or anything like that. Maybe that’s true. But maybe that’s not true. We don’t really know enough about quantum gravity to actually say. So the question about, is the planck size increasing? No, it’s not, as far as we know, that’s a formula in terms of these constants of nature and those constants don’t seem to be changing over time. There’s a different version of this question that is related to what Paul says, I’ve also heard that space might not be fundamental, are these two ideas contradictory? So if space emerges from quantum entanglement of degrees of freedom, of abstract degrees of freedom as we were talking about earlier, then there’s actually a really nice way of thinking about the expansion of the universe, namely that there’s a certain number of fundamental degrees of freedom that come together to form, to describe any one region of space, and what happens as the universe expands is not that more space is created, which would be this sort of a traditional classic way, classical way of thinking about it, but rather more degrees of freedom are becoming entangled. 1:18:44.6 SC: So you imagine, you know there’s a collection of quantum mechanical degrees of freedom, they’re all entangled with each other and making up space, so its spacetime, and there’s a separate bucket of degrees of freedom that are not entangled in some well-defined way, there’s subtleties and there’s math there, that I’m glossing over, but hopefully the image comes through. And that what you mean by the expansion of the universe is more and more of these unentangled degrees of freedom are becoming entangled and space appears to expand. I actually wrote about that in a paper that I wrote with some other collaborators called Quantum Circuit Cosmology. It was a very vague suggestion at the time, so we don’t have any tangible things to do with that picture, but it’s very attractive to me in terms of understanding the relationship between entanglement and expanding universes. And then for William’s question about, is the universe expanding? Are we expanding? Or is subatomic space expanding? No, that’s easy. No, it’s not, because and I think I have said this elsewhere, but the fact that space is expanding is an exaggeration. Parts of space are expanding, and in fact, when you go really deep into the math of it, it’s impossible to attach definite ideas of where space is expanding and where it isn’t. 1:20:03.0 SC: What it is possible to say is that this particular galaxy is further away from us now than it used to be. It takes longer for light to get there and to get back. But within a gravitationally-bound system, like a galaxy or a planet or a person, there’s no expansion of space going on; it’s only the regions in between the galaxies that are actually expanding. 1:20:26.6 SC: Okay, then there’s a bunch of questions about entropy, and I think I’m gonna group them into three groups, entropy questions. The first is these two questions, one from Ken Close, who says, “You’ve demonstrated entropy by mixing cream and coffee. There are interesting swirls before we reach maximum entropy, but I tried the experiment and about a day later, the cream had risen to the top, the mixture had been unmixed.” I don’t think you should leave out your cream and coffee for over a day, but I do know what you’re talking about, Ken. And then relatedly, Richard Graff says, “My understanding from discussions on Mindscape is that there are two definitions of entropy; one subjective that measures an observer’s knowledge of a system and one objective that measures the disorder of a system, how are these two definitions related?” 1:21:13.0 SC: So let me answer the second one first. There’s actually many different definitions of entropy, more than just two, but there is a fundamental distinction, and I wouldn’t put it exactly the way that you did, but I know why you’re putting it that way. 1:21:29.8 SC: So one way of thinking about entropy is, as you say, characterising our observer’s knowledge of a system. So, if you have a system that could be, given your knowledge, in any one of several states. You can think about the cream and the coffee, for example, you’re not knowing exactly where the individual atoms are, of water, or molecules of cream or coffee, etcetera, so you can rearrange them to get a system that looks the same, so you assign some probability distribution to all of the specific microscopic arrangements of the atoms, and then from that, there’s a formula that Boltzmann and Gibbs and others developed to calculate the entropy. 1:22:12.2 SC: Okay, so that is manifestly a definition that is talking about your knowledge. There’s another way, which is also subjective in a very real sense, but seems more objective, which is before you look at the cup of coffee, decide ahead of time what you’re able to see. Okay, so there are certain things you can observe about the system, certain things that are macroscopically accessible to us, and chunk up the set of all microstates into macrostates, saying that if two microstates are in the same macrostate then they must be observably indistinguishable, they look the same to you. Okay, so that once you do that, once you decide what your observables are, then you have a objective chunking of the space of all possible states into macrostates, and then what you can say is, you can assign an entropy, which is basically the size of a macrostate, it’s actually the logarithm of the size of a macrostate, given that you know that the system is in a certain kind of macrostate, you can assign an entropy to it, no matter what you know about it. 1:23:26.0 SC: No matter what you know about the microstate, okay? So it’s not that you don’t… Even if you know the microstate, if it’s in a big macrostate, you can still assign a large entropy to it. So that’s kind of quasi-objective, quasi-subjective. The subjective step was, you defined macrostates, the objective part is once you define that, there’s an entropy that you can associate with every microstate. How are they related? In the obvious way, namely that if your subjective knowledge of the system is precisely that it’s in a certain macrostate and nothing more, so then you put a probability distribution on all of the different microstates, which is equal probability for every possible microstate in that macrostate and zero probability for not being in that macrostate, and then the two entropy formulas coincide, they give you the same answer, so they’re compatible in that sense. 1:24:21.8 SC: Now, Ken’s asking about the cream and the coffee, and that’s because, it’s of course, very, very possible. You don’t even need to use cream and coffee, right? Try to mix oil and water. Do your best. They quickly unmix, much less than a day it takes, and that’s just because we’re being a little sloppy about the detailed interactions of the different molecules in the system, okay? So, for cream and coffee, it’s even more subtle, but oil and water work better. Oil molecules often have a hydrophilic component and a hydrophobic component. So given all of the different ways, given a certain energy in the system, it will often happen that secretly, the highest entropy states have the oil and the water separated from each other. Likewise for the cream and coffee, when you’re in a gravitational field and so forth and the molecules have different weights, it can be the case that the highest entropy states at a certain temperature and so forth are what the cream and the coffee are separate from each other, ’cause there are different kinds of things. 1:25:26.0 SC: So a better example would be if you had red balls and blue balls in a big box of gas with their masses and their interaction properties exactly the same, but just different colours, then they would really want to mix together. So, all of these typical things are just supposed to be ways of getting you to get your brain around the idea of things mixing together, but when you use as is often true in the real world, actual substances, then real physics can be more complicated than that. 1:25:56.5 SC: Okay, the next group of questions about entropy, Travis Hank says, “PBS Space Time did a video explaining how the growth of quantum entanglement drives the inexorable increase of entropy we see in the macroscopic world. My question is, why was there an initial state where the particles were not maximally entangled?” Nicholas Weinberg says, “Is there a close relationship between quantum coherence and the increase of entropy? Both seem to be related to the arrow of time and both seem to involve statistical system descriptions.” Michele Vincenzi says, “Did the Big Bang create the multiverse or was it already going? If it was already going, would it be right to assume that we got different big bangs in each universe or even universes with no big bang at all, or some completely different process?” And Cathy Sieger says, “In your conversation on the podcast with Max Tegmark, you both agreed on giving more credence to an Everettian multiverse than to a cosmological multiverse to exist. I wondered if when the Big Bang started off, the universal wave function also kicked in right from the start. Wouldn’t it be likely that the branching already affected very, very early states of the evolving universe and therefore in the process creating multi-verses in the cosmological sense?” 1:27:07.0 SC: So hopefully, the connection there is clear, those are four questions, and sorry about the length there. The first two were explicitly about quantum entanglement and entropy. The second two were about the initial conditions of the universe and those that’s very related to this question of entropy. So very quickly, Travis, you say that PBS Space Time did a video explaining how the growth of quantum entanglement drives the inexorable increase of entropy we see in the macro world. I don’t think that’s true. Actually, as I said just for the previous question, there’s more than one or more than two definitions of entropy, and indeed one of the other definitions of entropy is a strictly quantum definition, that John von Neumann invented. A fourth definition is from Claude Shannon in the context of information theory. But quantum entropy, sometimes called von Neumann entropy or entanglement entropy has to do with the entanglement between different quantum systems, and the formula for that entropy turns out to be the same as the formula that Boltzmann invented in the completely different classical context. 1:28:14.0 SC: But the point is that quantum entropy exists, it’s important for things like decoherence, this is Nicholas’s question. Quantum entropy of one system increases when that system becomes entangled with the outside world, and, of course, the most famous example of that is decoherence; when you become entangled with the environment, your total entanglement goes up, so your total entropy goes up. So to Nicholas’s question, is there a close relationship between quantum decoherence and the increase of entropy? Yes, that’s it. Quantum decoherence necessarily increases the entropy of a system as long as you don’t collapse the wave function and reset, that can lower the entropy. But to Travis’s question, that quantum entropy is a form of entropy, but it’s a different form of entropy than the classical entropy. 1:29:04.0 SC: When you’re mixing cream into coffee and you say entropy goes up or when you shuffle a deck of playing cards and you say entropy goes up, that has nothing to do with quantum entanglement right there, okay? That’s just a classical notion of entropy. So, these are different notions of entropy, both very important. And then your question about the initial state, you know, why did everything started out unentangled? This is related to the later two questions, and the answer is, I don’t know, no one knows. That is a big, big question. In fact, we’re gonna have a podcast coming up about this soon, but the point being that, that’s a major unanswered question in cosmology, why did the early universe had a special state which can be described as low entropy or maximally entangled? I would like… 1:29:48.9 SC: Now I have theories. I have proposed a model with Jennifer Chen, 16 years ago now, or something like that, where baby universes help understand it. But I don’t know if that’s true or not. It’s one of my favourite ideas, but it’s an open question, so to Michele or Michele, sorry, I don’t know, question. Would it be right to assume that we got different big bangs in each universe, and then Cathy’s question is related to that, is there a branching that gives you different parts of the multiverse. These are questions about the fact that the early universe had a special initial state that’s just an empirical fact. Mostly an empirical fact, there are some subtleties there, sorry about that. But it’s a fact, it’s a true thing. The initial state of our observable universe seems to be special, let’s put it that way. And the cosmological multiverse is a different conceptual idea than the Everettian many worlds. Everettian many worlds are caused by branching of the wave function, and there are literally full copies of the whole universe, that can happen right in this room right away, right when I measure a spin or something like that. Whereas the cosmological multiverse is an essentially classical concept that says that far away, there are regions of space where conditions look very different, so different that you might as well call it another universe. 1:31:16.8 SC: But you’re right, you’re both right in this implication here, which is that these two notions can be connected. Imagine that you have a wave function that branches, so you’re creating an Everettian many worlds kind of situation, but different branches have different local laws of physics. Now, normally that’s not what happens, that is not inherent in the many worlds interpretation at all. But it could happen if the events that are causing branching are the kinds of events that lead to the cosmological multiverse, namely bouncing around from one vacuum state in a landscape of possible vacuum states, in a more modern theory like string theory or extra dimensions or something like that. So this was an idea, and you can look up, I don’t remember what it was called, but I did a blog post about it years ago, this idea was actually pursued in two different papers that came out near the same time, one by Yasunori Nomura and one by Leonard Susskind and Raphael Bousso and both of them pointed out that you could sort of combine the many worlds idea with the cosmological multiverse and say that in different branches you had effectively different parts of the cosmological multiverse. I think that’s right, that sounds right to me. 1:32:33.0 SC: It didn’t take the world by storm, because it’s not clear what to do with it exactly, it’s not clear that you solved any problems by pointing this out. It seems to be a true feature, but it doesn’t help you explain why the early universe had low entropy or why our universe looks the way it does. But maybe it’s something to keep in mind, when you’re thinking about cosmology and entropy and the Big Bang and the multiverse and all that stuff. 1:32:58.7 SC: Okay, I have one more group, the third group of entropy questions. One is from Nick G, who says, “Where does entropy and the Second Law of Thermodynamics fit into the fundamentalness hierarchy? Is it just an emergent phenomenon that isn’t exactly written into the source code of the universe, or is it something deeper and more fundamental?” And Paul Toric says, “Nick G’s question about where entropy fits into the fundamentalness hierarchy inspires my question. Do we even need a fundamentalness hierarchy or is it just a matter of which order or presentation of facts we find explanatory, and or which statements of laws and boundary conditions are simpler, or is the latter all that is meant by fundamental?” 1:33:40.3 SC: So let me talk a little bit about my view of this fundamentalness hierarchy, and then hopefully that will eliminate both questions. The idea of the fundamentalness hierarchy is an old one, at least August Comte, who I think I was mostly interested in Sociology, talked about this over 100 years ago. And it’s also very natural, we sort of take it for granted all the time. You’re a human being, you’re an individual, an organism all by yourself. You are a part of bigger structures; you’re part of a society, you’re part of the whole Earth. And you’re also made of smaller structures, you’re made of organs, your organs are made of cells, the cells are made of molecules, made of particles, etcetera. So there’s kind of an obvious hierarchy, and I do think it’s more or less subjective in the big things are made of little things in the universe as we see it. 1:34:29.8 SC: Now, I don’t think that that’s necessary. If we sit back and think about all the different possible laws of physics that we could imagine, ways the universe could be put together, is it necessary that no matter what kind of source code we started with, would we always end up with some kind of fundamentalness hierarchy? No, I don’t think that’s necessarily true. It’s a feature of… There’s a couple of ingredients that go into it; the most important of which is locality, that there is something called space, that there are objects located in space and they tend to bump into each other when they’re nearby and not when they’re far away. That’s the fundamental feature of the world. It didn’t have to be there, there are other ways of thinking about worlds which wouldn’t have that feature, but it enables this feature of conglomeration of small things into bigger things. 1:35:24.6 SC: But there’s an additional feature over and above that, which is that there is something real about treating collections of many small things as things in and of themselves. So in other words, we take the world and we divide it up into things, and that’s sort of a… Not an innocent move, you can question, “Why do you divide up into this set of things?” So I forget exactly the example that I’ve heard, but the collection of atoms in your body come together to form you, and it makes sense to talk about you. Why? Well, because it has some causal role in the world. I can make statements like, “You went there, you did this. If I tell you this, you will act in a certain way.” There’s a higher level theory of where you fit into things, whereas, if I make some collective object consisting of your left foot and this cup of coffee and that oak tree out there, that’s an object I can define, but it doesn’t have any role in the world, there’s no theory in which that collection of things has an autonomous existence or useful way of talking about it. 1:36:38.2 SC: So there is something real. Even though it’s convenient for us to talk about these higher level emerging phenomena, there are right and wrong ways of doing it, there are better and worse ways of dividing the world into objects. And that goes back into Nick’s question, the second law of thermodynamics is not fundamental. If you were Laplace’s demon or the quantum mechanical equivalent thereof, if you knew exactly the micro-state of the universe, you wouldn’t have to do any of this course graining or thinking about our ignorance, you wouldn’t have any ignorance, so you wouldn’t have any entropy to deal with. But, the fact that there is a useful concept called entropy, that has causal explanatory power, we can explain things going on in the universe by talking about entropy increasing, etcetera, means that there is some reality to that concept, so it’s not fundamental, but it is real. That’s my point of view on emergent phenomena more generally. So hopefully, that helps with those questions. 1:37:40.6 SC: Okay. Enough about entropy there. Alison says, “Could Krasnikov tubes, permanent superluminal tunnels, in principle be used to construct galaxy-sized telescopes?”? 1:37:55.6 SC: I don’t know, I guess, maybe. I presume I had something in mind to say about this question, when I kept it in the list of things I’m trying to answer. But this is the kind of thing… Maybe this is what I wanted to say. The space of speculative ideas that naturally suggest themselves to human beings, is very different and potentially unconnected to the space of speculative ideas that could be true. In other words, the way I think of it is, think about old science fiction, 100-year old science fiction. They got some things right. Space travel is true, we all now have portable communication devices. But they missed other things. They missed the fact that we don’t have faster than light travel. They didn’t miss that, but many of them just let themselves, helped themselves to faster than light travel, whereas no one really invented the internet, or something like that, and very few went into sort of modern synthetic biology or things like that. So our ability to predict real world scientific advances is uneven. 1:39:09.9 SC: Why? Well, one of the reasons why, is because the kinds of advances that it’s easy to predict are taking things we already do and imagine we can do them better. We already, 100 years ago, could travel, we could fly, we had cars, we even had airplanes. We could make small things that go through the air, so it’s just easy to extrapolate them to faster things and things that go further away, etcetera, whereas a conceptually new category like the Internet, is harder to imagine. So when you say permanent superluminal tunnels, they’re not gonna exist, honestly, in the real world, that’s just not going to happen. If it did… But they’re the kinds of things that we are tempted to imagine, because we can already go pretty fast and it’s easy to imagine going arbitrarily fast. I don’t exactly see… You might be getting at some issue with a telescope question, I’m not quite sure why a superluminal tunnel would help us build a telescope, but I just didn’t wanna say, that’s not the way that I would spend my money, honestly, if I were thinking… Or even spend my time thinking about possible future technologies. 1:40:21.6 SC: Okay, Michael Lesniac says, “I first became familiar with you on science Channel shows like Through the Wormhole, and I’m curious, how did the interview process for shows like that go, who contacted who, and how come I haven’t seen you on any more recent shows like that?” 1:40:36.5 SC: Well, for the recent shows question, I think that… Maybe this actually goes to the first part of the question, the process is very informal, about being contacted. Someone looks you up. In fact, yeah, let me back up and explain how these shows are very often made. Different shows are made in different ways. Like a PBS show, a NOVA show will very often, be based on a book or something like that. When Brian Green does his Elegant Universe books or something like that, and it turns them into a NOVA series, he’s very involved in what is going on. But, for History Channel shows or Science Channel shows or Discovery Channel shows, the process is different and often, it’s not a very good process. 1:41:16.1 SC: Through the Wormhole was one of the better examples, but here is the basic process: The TV network hires a production company to make a show, Through the Wormhole, or The Universe, or Mysteries of the Universe or whatever. The production company hires a director or a producer sort of be in charge of the particular show, and that director or producer hires writers. What does that mean? You hire some people who are TV writers, they work for the production company, and their job is to come up with an episode and maybe there is some brainstorming among some people, about what the topic of the episode should be, let’s say, the black hole information loss puzzle, and then this writer or a production assistant or some other sets of people, the producer themselves, goes on to internet, Googles around, looks for experts, especially looks for experts nearby. This is why living in Los Angeles is very helpful, if you wanna be on these TV shows, and says, “Who can I talk to about this?” 1:42:20.5 SC: And so, they will sketch out from Wikipedia and other things, other videos they see on YouTube, they’ll sketch out their idea of what the good things to talk about are, and get some ideas of who they might wanna talk to, and then they might call you up and interview you and decide whether they wanna have you on camera or not. But The point is that, none of the people involved in that are scientists. They interview the scientist once they get on camera, but the writing and the organising, and deciding what to do on the show, is rarely guided by people who are knowledgeable about science. And so, the individual segments are often perfectly respectable, but the overall vision of where to go is not there because there’s no one really scientifically informed guiding it. They don’t hire a scientist to do that kind of thing, and that sometimes leads to embarrassing things, like I was once doing a demonstration for one of these TV shows about dark matter, and I think I told this story before, but I was asked to spin a light bulb above my head, and they were gonna put special effects in there, so the light bulb would light up, etcetera, and this was supposed to be dark matter. 1:43:33.4 SC: And so, I asked, “Am I representing the dark matter and the thing zooming around me is supposed to be like a star or something that is feeling my gravitational field?” and the people filming me, honestly didn’t know. There had been so many steps in between, when someone came up with the idea for this analogy, and when we got to filming it, the meaning of it had been lost, and that can happen sometimes. But in other times, the shows turn out really well. If you get the impression, I think it would be better if a lot of these shows took the scientific side more seriously and actually let scientists be involved, but that would involve money and things like that. So who knows whether that will ever happen. 1:44:13.8 SC: Why haven’t I been on any more recent shows like that? I have been on a couple, but not that many. Partly, it’s just a small number of statistics about when you get invited, partly there’s a pandemic, partly I’ve been busy, so sometimes I’ve had to say no when I get invited to do these things. It’s a weird thing. You reach a lot of people, but you don’t have a lot of control over what’s being said. As I get older and grumpier, I like to have more control over what’s being said. 1:44:39.8 SC: Paul Hess says, “Is a nation of millions of people conscious?” 1:44:45.0 SC: Rough answer is no, I don’t think so. But it’s an interesting question, and you can ask it at different levels of precision. I think that if you look into things like… What is it called? Integrated information theory, IIT, this idea of Giulio Tononi and collaborators, where they tried to actually answer this question precisely. They propose a measure that is quantitative, they can sort of pinpoint the level of emergence at which you would call something conscious, the idea being, let’s say you have a mob of people, if they’re just acting randomly and differently, then each individual person is conscious, but sometimes you get mob behaviour, where in some sense it makes more sense to attach the locus of consciousness to the mob, rather than to any individual inside it. 1:45:33.3 SC: I don’t think that that’s very helpful in the case of a nation of millions of people. We do it all the time, we do the equivalent thing because we attach beliefs, desires, attitudes, psychologies to countries of people, but it’s often just a short hand for what should be a more nuanced way of talking about things. We don’t have the once and for all definition of consciousness, so I’m not quite sure if we can carefully answer this question at all, but I think that roughly speaking, my answer is tentatively no. 1:46:06.4 SC: Okay, Jeff B asks a priority question. Remember, the priority questions are, once in your life, you get to answer a priority question. I promise to answer it to the best of my ability. Sometimes my ability is not very good, so be careful about how you ask your priority question. So Jeff’s question is, “The question boils down to how a waving field ends up getting detected as a dot on a detector screen. Your previous answer is basically local interactions plus decoherence. This makes sense to me, but only under the assumption that the dots are somehow given special treatment during wave function branching, otherwise, I could imagine the wave function branching, so that each branch saw something other than a dot appear on the screen, a circle, triangle, whatever. My understanding is that these dots are special, because they are pointer states that are stable under decoherence. So my question becomes a bit meta, why are these pointer states… Why are these the pointer states and not something else? Our world would probably look very different if the wave function branched differently, but it seems that there has got to be some underlying ontological reason for why it branches in this particular way.” 1:47:09.5 SC: So I think the answer is, it is because of locality plus decoherence, but you’re right, that there’s more steps to that story a little bit. You have a gun shooting electrons or something like that at a screen, in principle, the wave function is spread out over different points, but in fact, when you see the screen light up, it’s at one point, it’s not at either the whole smeared wave function, nor is it at some weird shaped sub-system, like a circle or triangle or whatever. Why is that? 1:47:41.8 SC: Basically, the reason is that what pointer states are, are parts of the wave function which have the property that once they’re there, once you’re in them, they don’t keep getting extra entanglement with the rest of the world. Remember, entanglement is a special kind of quantum mechanical interaction, like if you take a basketball and bounce it on the floor, the basketball does not become entangled with the floor, because the whole basketball interacts with the floor in exactly the same way. If you had a basketball that was in a superposition of, let’s say, spinning and not spinning, and then you bounce that on the floor, then the spinning part of the basketball wave function would bounce in one direction, and the non-spinning part would bounce in another direction. So those two parts of the wave function would separate, they would decohere because they would become entangled with the rest of the world, that would be an interaction that entangles them. 1:48:40.6 SC: So the point about the electron or whatever hitting a detector screen, is that… Think about the ways that that part of the wave function interact with the rest of the world. The electron interacts with the detector screen, and then it leaves a dot or it leaves an impression that you can see, by looking at it, there’s some change in the colour or the brightness or whatever of the screen. So you can imagine tracing out a circular pattern on the screen, but in fact, all of those parts of the circle interact differently with the world, they all give out what amounts to a photon, but photons moving in different directions, ’cause they’re coming from different angles on the screen. All of the parts of the wave function where the electron hits in the same point, interact in the same way with the rest of the world. That’s why it is a pointer state. 1:49:35.3 SC: A much more vivid example is something like Schrödinger’s Cat, where it’s alive plus dead, or awake plus asleep. If you wanna buy t-shirts from the merch store at Mindscape, you can get an awake plus an asleep cat on your T-shirt. The awake and asleep part of the cat’s wave functions interact differently with the rest of the world, because they’re in different macroscopic positions. So it’s that position that comes in in all the cases, whether it’s the basketball or the cat or the electron hitting part of the screen, different positions where the interaction happens, lead to different kinds of entanglement with the rest of the world, and that’s why pointer states tend to be the subsets of the wave function that is sort of macroscopically coherent, that describes something definite in the space. And in the case of an electron hitting a screen, that’s gonna be hitting at a point in space, hitting at a point on the screen. 1:50:34.0 SC: Claudios Limavitz says, “I read a biography of Fred Hoyle, and he strikes me as a fascinating figure. He seems to have made important contributions, especially on Nucleosynthesis. He also had controversial, later debunked views such as the steady state universe. How is Hoyle perceived in the physics world? An underrated genius or a crack pot?” 1:50:52.1 SC: I think he’s perceived as a rated genius. His insistence for the steady state universe was quirky and lost him some favour, but look, there were other people who hung on to that steady state universe long after Hoyle went away, and they definitely should have known better. If you’re sort of the originator of an idea, then I think a lot of people will cut you some slack for hanging on to that idea a little bit longer, but they would prefer it if you changed your mind, that would be better. But I do think that everyone agrees that Fred Hoyle did a lot of great things in Theoretical Physics and Astrophysics, especially nucleosynthesis and the origin of the elements. There is a book that just came out, by Paul Halpern, contrasting Fred Hoyle and George Gamow. So George Gamow, in some sense, was the champion of the Big Bang version of the story, and Hoyle was the champion of the steady state version. They all had collaborators, and the collaborators are important, so I don’t wanna brush them apart, but these were the two big personalities in this game. 1:51:58.1 SC: And they were both part right and part wrong. Ultimately, on the Big Bang story, Gamow was right, but then you want to explain where the elements come from. The Big Bang didn’t help that much with that. It gets you helium, but it’s not enough, and then stars and supernovae are necessary to get the heavier elements, and some people, rightfully hesitated to say, “Well, the real story involves a little bit of this and a little bit of that.” If you remember the podcast we did with Simon DeDeo, talking about why certain kinds of explanations are favoured over others, the ability of one thing to explain many things, is rightfully thought of as a good feature for an explanation to have, and this makes people a little bit leery, weary, I should say, a little bit weary, when the right explanation seems to involve disconnected unrelated things. But that’s really very often true in the real world, that you need to invoke disconnected unrelated things to explain everything you see in the world. And for the formation of elements in the universe, that is very, very much the case, but people appreciate, I think, these days, the contributions of Fred Hoyle as well as Gamow and others. 1:53:11.8 SC: Peter Solfest says, “After re-reading The Big Picture, I was wondering about strong emergence and quantum field theory. Is it theoretically possible or mathematically consistent to construct a quantum field which only interacts with other fields if they are sufficiently complex?” 1:53:28.2 SC: So I’m glad you’re asking that, because I’m literally writing a paper about it right now. To be honest, not going into much details, but I’m finishing up a paper, it’ll appear soon, on Consciousness and the Laws of Physics. And it’s always dangerous when physicists are talking about consciousness, I get that, but I’m not saying that much actually about consciousness. All I’m saying is something I’ve said before, but I’m saying it in a little bit more detail, namely, whatever is going on in consciousness, don’t try to explain it by changing the laws of physics. That’s not the way to look. The laws of physics are pretty well established and we know a lot about them. Don’t add things to it, to change the laws of physics, just ’cause you wanna understand consciousness. There’s other strategies that are much more promising. 1:54:12.4 SC: And in particular, there’s this question of locality and strong emergence, and basically, let’s say it this way, we know that even if you think there are other things in the universe, there are atoms in your head, there are electrons and protons and neutrons in your brain, and what happens to those particles in your brain is certainly somehow related to consciousness. Maybe it’s the whole thing, maybe it’s only a part of it, maybe it’s epiphenomenal, but it’s somehow related. And so, one attitude is, “Sure, you have a theory, the core theory, which tells me, if I have an electron surrounded by certain other fields, how it behaves. But maybe that works at CERN, or maybe it works at your laboratory table, but who’s to say that’s the correct set of rules for describing what an electron does, if it’s in a human brain, if it’s embedded in that very complex system?” 1:55:13.2 SC: Well, the answer is, everything we know about the rules of effective quantum field theory, says the electron could not possibly care less, whether is embedded in a human brain. Electrons interact locally. The only thing that affects what an electron does is the values and the derivatives of other fields at the same exact location as the electron. And in fact, the other atoms, the human brain seems very complicated and dense, but the particles in the human brain are kind of far apart from each other by particle physics standards. So we would expect… And it’s just an expectation, it’s not a law of nature, but our expectation is, that the behaviour of those electrons is perfectly local. So if you want to come up with a model, where somehow the dynamics of individual electrons change because it is in a complex system like a human brain, you’re welcome to trying. I’ve tried, not ’cause I think it’s true, but because you sort of try to shoot it down, and I was not able to either come up with a model where it worked or to come up with a proof that you couldn’t come up with a model where it worked. 1:56:21.0 SC: I can say with confidence, there’s zero expectation of anything like that happening, except for your desire to make the human brain be somehow special. So I wouldn’t worry about it too much, but I can’t tell you that it’s absolutely impossible. 1:56:37.4 SC: Andrew J says, ” Laplace’s demon has perfect knowledge of all micro-states for a single instant of time, as well as the full understanding of the true laws of physics and can therefore predict the future and retrodict the past. Is the demon then limited to predictions, computations only for micro states? If that’s the state… If that’s the case, then isn’t the demon essentially clueless about predicting any future that might be meaningful to people?” 1:57:02.3 SC: This is a slightly deflationary answer, but I don’t know what Laplace’s demon can do or cannot. The demon all the… He knows the exact micro state of the universe and the laws of physics and has the computational power to both predict and retrodict. Is that all the demon knows? I don’t know, I don’t see what limits the demon. Presumably, if the demon has that much computational power, which is a lot, maybe they’re also able to figure out what the emergent properties are. There’s no obstacle to that, in principle. So the demon was made up, it was a thought experiment, you could make up a version of the demon that’s able to do that or a version that isn’t. 1:57:41.1 SC: Maybe the substitute part of the question is, could you imagine knowing the whole micro-state, could you imagine in principle, knowing the whole microsite, and not knowing the sort of ways to chunk it up into emergent higher level things. If that’s the question, then the answer is yes, absolutely. It’s like if someone gives you… Well, if someone takes, I don’t know, Mac OS, an operating system, or Linux or something like that, and translates it into binary, the assembly code into binary, and then prints it out as a set of ones and zeros and hands it to you without telling you what it is. Could you figure out what it is? I’m predicting, the answer is no. Even though all the information is there, you don’t have the ability to know, “Oh yes, that’s Mac OS, 2018 version,” or something like that. There’s a whole another layer of conceptual insight that it takes to do that. So if that’s what you mean, then we can at least imagine a micro-state version of Laplace’s demon without implying a macro state version. 1:58:40.3 SC: Warren Harris says, “In your last AMA, you invoked David Lewis’s Possible Worlds to explain your position on the principle of sufficient reason, saying that the fact that we live in one world, rather than the other, has got to come down to a brute fact. I found your response surprising because, Lewis’s modal realism provides a reason for apparent brute facts and that reason, self-locating uncertainty, is that very same, invoked by yourself in answers to, ‘Why do we find ourselves in this branch rather than the other in Everettian quantum mechanics?’ The anthropics that provide the reason for apparent brute facts in the MWI, are the very same mechanism by which brute facts can be explained in Lewis’s model, which is an example of a metaphysics that at bottom, satisfies the principle of sufficient reason by being maximally symmetric, not reifying one brute fact over any other. Can you expand on why you don’t accept brute fact explanations of chancy outcomes in quantum mechanics, but do accept them in a seemingly similar situation?” 1:59:36.0 SC: So there’s a lot going on here, and I like the question more than I think I have a great answer to give. I want other people to think about the question, if it makes sense. I am very fond of self-locating uncertainty, and also David Lewis’s idea. I think it’s his idea. I’m not even sure it’s his idea, that… Well, his way… He is the one… He’s the person from which I got the idea of thinking about uncertainty about the laws of physics, as self-locating uncertainty in the space of all possible worlds. There’s just a way of thinking about the fact that we don’t know what the laws of physics are, we can imagine different worlds with different laws, and then saying that we don’t know what the laws are, is the same as saying, we don’t know which of those possible worlds we live in. 2:00:27.9 SC: But there is an important difference. So I think that… If I get into the nitty gritty of your question, I don’t understand the claim that Lewis’s model satisfies the principle of sufficient reason. I think it depends on what you mean by a reason. What can I say? Even in the quantum mechanics case, where I think I do understand things, there is a version of me that sees the electrons spin up, a version of me that sees the electrons spin down. The one of me that sees electrons spin up, do I get to say, “There is a reason why I see it spin up, and the reason is, well, because there’s another one that sees it spin down and they’re symmetric.”? I don’t know if that really qualifies as a reason, maybe it does, and maybe that’s an ambiguous question to which there’s no perfectly good answer. So maybe that’s part of the set of assumptions that we would have to dig into here, to give you a satisfying answer. 2:01:17.9 SC: But the other thing is that I’m not a modal realist, I am an Everettian, and I do believe in the different branches of the wave function, but I don’t believe in the reality of all the possible worlds. I’m happy to use the set of all possible worlds as a way of thinking through our uncertainty about laws of physics and about questions of causality and things like that, but I don’t therefore, attach to them, reality. So to me, it seems most likely there is a universe, even if it’s a big Everettian multiverse, it’s still one universe in some sense, with laws of physics, and the question about why those are the laws of physics rather than others, does not come down to self-locating uncertainty in a set of real, actually existing world. It’s a different kind of thing. So I don’t have the similarity, the analogy there, the parallelism between modal realism and Everettian quantum mechanics. 2:02:12.3 SC: For those of you who are interested, Alistair Wilson, who is a philosopher, has tried to develop that parallelism, he tries to claim that we can think of Everettian branches of the wave function as Lewisian possible worlds. I don’t really get on that train, but it’s an interesting way to think about the question. 2:02:33.4 SC: Saraz Rajon says, “The universe is amoral, and the prospect of a no consequence after death could either be liberating or disappointing. Liberating as in, this is the one life we get to live, so live it as you please, satisfy your brain’s whims and fancies. Disappointing as in no matter what legacy or goodness I leave after my death, it is not going to be of any use to me, I’ll be non-existent. Do you have a different take on this matter?” 2:02:57.4 SC: I think neither one of those is exactly how I would put it. The fact that life stops at death and there is no afterlife, is going to be disappointing to those who place the value that they have in their lives, on something that happens or continues on after their death. You’re not here anymore, so as far as you’re concerned, there’s no enjoyment or disappointment to be gained in what happens after you die. So having a legacy and things like that, there is a… Well, I was gonna say it’s irrelevant, but there’s a crucially important exception to that, which is that you might very well now, care a lot about the prospect of doing something that even lasts after your death, and that’s perfectly valid. So, to take pleasure in establishing something now, that will only pay off after you’re dead, there’s nothing wrong with that. Even though you won’t be around to appreciate it, you’re appreciating it now, that’s a perfectly 100% okay. But if you really thought that somehow, you needed your life after death to care about what happened after you died, then I got some bad news for you. That’s not gonna happen. 2:04:11.4 SC: Now on the other side, you say, “This is the one life we get to live, so live it as you please and satisfy your brain’s whims and fancies,” that’s just a non sequitur in my mind. There’s only one life we get to live, therefore live it as you please, it just does not logically follow. You need some extra premises that you’ve smuggled in there. I think you only get one life to live, and therefore what you experience in this life and the impact you have on other people’s lives, are the things that matter. The things that matter are not the experiences you will have after death, because there are no such experiences, but the experiences you have here on earth matter, they go far beyond your brain’s whims and fancies. That’s a sort of an intentionally belittling way to put it. Everything good that you do in the world and experience in the world, are things that are happening here in the world, right now. And so, that’s okay. My attitude is just, that is what matters, the stuff that you see, the stuff that you experience, this is a perfectly legitimate basis for caring and mattering in the world, and it is in fact what actually exists, which is good. 2:05:30.9 SC: Sam Hartzog says, “If memory serves, I was introduced to Robert Forward’s Dragon’s Egg novel by your recommendation in some form or another. If so, thanks. I love this book.” 2:05:41.7 SC: Probably that’s true. I’ve definitely recommended that. It’s not one of the world’s most famous science fiction books, but I like it, so I do sometimes recommend it. The question is, have modern field theories ruled out the complex ultra-fast nucleonic interactions that give rise to the neutron star’s chemistry? 2:05:58.9 SC: So in the book, there are living beings that live on the surface of the neutron star, that live in a sort of organic life that is not based on chemistry, but more nuclear physics. So I don’t know, is the short answer to this question. I haven’t read the book in a long time, and I haven’t thought about life on a neutron star, in any careful way. My simple guess would be, it is completely incompatible with what we know about nuclear physics, and the basic reason is, there’s not a lot that can happen in nuclear physics. The reason why Chemistry is interesting, is that you can make atoms, but you can hook atoms together in arbitrarily complicated ways. The centre piece of life is DNA, and the great thing about DNA is, it’s an aperiodic crystal, as Erwin Schrödinger predicted even before it was discovered, which means it can be as long as you want without ever repeating itself. Information can be contained and passed down from generation to generation in the form of this quasi-stable molecule, and there’s no limit on how complicated in principle it could be. 2:07:13.8 SC: Whereas, if you’re just making things out of neutrons or even neutrons and protons, good luck with that. There’s not arbitrary complexity that you can get there, as far as anyone knows, as far as I know, as far as anyone that I know knows. So my guess is, that’s just not possible, that’s just not how nuclei work. But I haven’t thought about it very carefully. 2:07:34.4 SC: Jim Murphy says, “You’ve said before that you’re convinced space is not fundamental, but less convinced about time. How can this be the case, if space and time are just two aspects of the same thing?” 2:07:43.7 SC: Well, maybe they’re not. That’s how. So the reason why that’s a short… Not very clever answer, but the reason why I wanted to talk with this question is, it raises an interesting issue for people who wanna develop theories beyond the currently existing theories, and this is part of what we talked about with Lis Mullen in the podcast. There are things we know, things we don’t. How do we choose what kinds of directions to move in? So something that is a major triumph of 20th century physics is, space and time are just two aspects of the same thing, space time. And what I’m proposing, not just me, but what is being proposed, is that we can think about emergent space time in a way that treat space and time differently, just like Newton would have treated them differently. 2:08:33.9 SC: So in some very real sense, it’s a step backwards. And you can ask, “Is that a strike against this approach?” And I think the answer is yes. I think that if there were no other strikes, that argument, you’re losing some progress that had been made by physics, is a perfectly good reason to be sceptical of this approach. So if this approach is right, not just my approach to merging space time, but if the specific version of it, in which time is fundamental, but space is not, if that’s true, then time and space are fundamentally different in some way, and the fact that they seem to be two aspects of the same thing is kind of an accident that emerges in the big macroscopic low energy universe. Is that kind of a robust, natural thing to happen or is it a special weird thing? That, I don’t know. But it is part of the challenge of this approach, that you would have to explain why it emerges in that way. 2:09:31.5 SC: Alex Borland says, “I have a question about the simulation hypothesis. Imagine that humanity managed to simulate a universe complex enough to contain intelligent life. If that intelligent life started to study their universe and tried to discover its physics, at what point would you give them credit and say they had cracked its fundamental laws? Would it be once they had figured out all the equations our super-computer was running, or would they have to discover a deeper level of reality, such that the programming language they were written in, or the hardware they were running on, etcetera?” 2:10:01.9 SC: And actually, now that I think about it, I’m gonna group this question together with another one from Krather Luca, who says, “Last month, in your AMA, you mentioned that eventually, our knowledge will bottom out. I interpreted this as saying you eventually… That eventually, progress stops because we reach some cold hard facts. I agree, there probably is a place where things bottom out, namely, reality. However, how can we ever hope to know that we’ve reached such a place?” 2:10:26.1 SC: So Alex’s question is, if you’re in a simulation, how do you know that you’ve reached the bottom level. Krather Luca’s question is, how do we know if we’ve ever reached the bottom level? 2:10:34.7 SC: Yeah. We don’t. We’ll never know that kind of question. I think that this is a… No one ever promised you a rose garden, when it comes to the science. So I do think that given that I’m not a modal realist, given that I think that not all possible universes really exist, there are some fundamental features of our world, and by the way, when I say that all of these things are very tentative and I’m not absolutely wedded to these things, I’m willing to change my mind about them, but I’m just saying where I am right now. There are some facts about the universe that are explained by nothing deeper than, “That’s the way it is,” brute fact, cold, hard fact. But we don’t know which features are like that and which are not. The mass of the electron, actually the mass of the electron is dimension-dependent. So the ratio of masses of the muon in the electron, that’s a dimensionalist number. Okay, is that an accident or is there some deep explanation for it, for example? Why are the fundamental laws of physics quantum mechanical? Is there a deep reason for that or is it just a thing that we just have to accept? 2:11:49.6 SC: I don’t know, and I don’t think that there’s anything wrong with not knowing or even knowing whether you will never know. So the attitude that I would advocate taking is, it’s perfectly okay to try to answer those questions, to try to say, “Okay, why quantum mechanics? Why relativity? Why the mass of the electron and things like that? Why does the universe exist at all?” What’s not okay is to insist that there must be an answer that would make you happy in terms of some other features of the universe, rather than saying, “One of the possible options is, that’s just the way it is.” So I would advocate keeping the cold hard fact option as an option, while not necessarily ruling out the search for other possible answers at the same time, and that there’s no point at which you say, “Okay, now I know for sure.” But that’s just science, that’s just how science works, there’s never a point you reach in which you can say, “I’m completely sure.” 2:12:49.0 SC: Alright, now grouping together a few more questions, ’cause these are all in a very different… Now, for something completely different. Questions about the podcast. So Jesse Rimler says, “I’m continually impressed by the variety and quality of your guests. Could you tell us a little bit about your methods of discovery? As an author, do you receive advance copies of books?” Russell Wolf says, “In a recent AMA, you talked about how you always take recommendations for podcast guests, but don’t like to make any promise about whether they will appear. I was wondering if you could say anything in aggregate, about how often a listener recommendation results in a guest that you otherwise weren’t considering?” And finally, Seth says, “Would you ever consider a podcast where you go into detail about your day-to-day process for producing a podcast? I really love how accessible you’ve seen and how you don’t make it seem like what you do is magic.” 2:13:37.6 SC: Yeah, what I do is certainly not magic, and in fact, Seth, I don’t think it’s that worthy to go into the podcast and talk about my day-to-day process ’cause it wouldn’t take that long. I will tell you what my day-day process is, in as much detail as anyone would ever want to hear. I do have to find guests, so this goes to the questions asked by Jessie and Russell. The way I find guests is a wide variety of different things. Sometimes, people, Patreons or people on Twitter, or comments on preposterousuniverse.com/podcast, or on the Reddit, there’s a subreddit, Sean M Carroll subreddit, they make suggestions and I take them all seriously. If I have never heard of the person, I’ll Google them or whatever. I get a lot of suggestions, so if your suggestion is not followed up on, don’t feel bad, I can’t possibly do everyone. 2:14:30.5 SC: Sometimes I get a suggestion, I do invite that person and they say no. And so, I don’t ever want to… This is the biggest single reason why I don’t want to say who I’m considering, because I totally respect people’s decisions not to appear on the podcast. I don’t wanna sort of shame them publicly for not appearing on my podcast. That’s not gonna happen. They have every right, and anyone… If I ever invite you to be in the podcast, feel free to say no, I get it. I say no all the time, when I’m invited to be on other people’s podcasts, especially since I have my own podcast now. There’s only so many… So large a fraction of my time that I’m willing to spend, talking on podcasts, and 98% of it is talking on my podcast, these days. 2:15:13.8 SC: But there are other places where the guest suggestions come from. Yeah, you’re asking about, do I get books. So many books. Both ’cause I’m an author and I have the podcast and I have the blog, and my wife, Jennifer, is a science writer, we get a lot of free books, and this makes me feel very bad to say it, because when I was in college or grad school, I would just spend hours in the bookstores, lovingly looking over all these books that I will never own, and now, books just appear at my house and I have to actually give many of them away without ever reading them ’cause there’s too many. I didn’t ask for them, they just appear, and so, that’s a terrible position to be in, but so be it. 2:15:57.3 SC: So, very often… I don’t know why people send me unsolicited copies of books, you can just email me and say, “Would you like a copy of the book?” That’s a much more cost-effective measure. Many times, if a book is coming out, now that I have a podcast, the PR person, either for the individual author or for the publishing house, will email and say, “Would you like to consider this person for the podcast?” or the author themselves will email me saying, “Would you like to… Could I appear on your podcast?” And I love it. Both cases, whether they themselves or someone in their orbit, suggests someone for a podcast, I like being suggested people for the podcast, as long as they do not mind if I say no. Again, I get way more suggestions than I can possibly take up. So people who wanna argue with me when I say, “Sorry, it’s not fitting into my schedule right now,” and they say “No, but did you consider this?” That gets annoying, but… And usually, that very rarely happens, actually, most people get it, that there’s a limit on how much you can possibly do. And in fact, it’s not too surprising, in the case of a long existing podcast. 2:17:06.9 SC: We’re actually, now that I realise it, we’re almost at exactly at the three-year mark for Mindscape, so we’ve existed for three years. Yeah, yeah, it’s our birthday. I have invited people to be on the podcast, and they’ve said no, and then a year later, their publicity person says, “Would you have my author on your podcast?” and usually, I will say yes. But I get it. Life, circumstances change. Sometimes people say, “Well, I ordinarily would like to. But right now, it’s not a good time.” All that is fine, I totally get it. And other times, I just wanna have a topic. There’s some… A person or a topic that I’m interested in, so no one suggested them, and I go out and find them and have them on the podcast. So all of these work. 2:17:51.3 SC: As I’ve said before, I want people who have something interesting to say, something that is worth listening to. I’m not in the debunking business, I don’t wanna debate or argue with people. Even if I disagree with them, I wanna hear what they have to say and learn from it, and the audience also. But within that very, very broad category, I want diversity, I want all sorts of different people, I want people in different topic areas, I don’t even wanna have two of the same field in a row, if I can avoid it. 2:18:18.0 SC: Sometimes you can’t avoid it, and sometimes it works out okay, but I want a diversity of fields, a diversity of perspectives; I want some old people, some young people, some famous people, some not so famous people. The other thing is, you do want people who can be good on a podcast, and that’s a whole another skill set. Like I said, in another context, you can be really good at the intellectual side of things and not on the public side of things, so I want people who can articulate what they’re doing, and so usually it’s easy to… It’s possible anyway to suss out that criterion, but not always. And so, I think… Look, overall, I’ve been very, very happy with the total set of people that I’ve had on the podcast. I look back at the list of people we’ve had, and some have been more successful than others, but I’m like, “Well, look at all these people who were here on this podcast.” That’s pretty good. So happy with that. 2:19:09.1 SC: Okay, then to continue on to the sets of question about producing the podcast, we set up a time. I do try to make sure that they have a microphone and annoyingly, there’s this thing where the programs I used to actually do the recording, which is usually Zencastr, but there’s some other programs, this is like a website that I send the guest to, and I also go on, then we can talk and record, they keep fiddling with it and removing browsers that work. So it used to be that Zencastr worked on all browsers. Now it’s only on Chrome, which is very annoying. 2:19:44.8 SC: Anyway, once we can all set that up, set a date, send them the Zencastr link, we both appear and we talk and I record it, and then I record it into Audacity, which is, so I… Well, I record it… That’s not exactly true. I download the audio files from Zencastr and then I import them into Audacity and edit them. So Audacity is a free audio editing program. And I don’t do much editing, I try to clean up, I know that not all the audio is equally good, and usually that’s just because the person on the other end is in a very echoey room or a very noisy microphone or something like that, and it’s… You try to clean it up but you can’t. It is very, very difficult. But I do a little bit of that, and then I remove the coughs and whatever to the extent that I can, and then I record an intro and I upload it and it goes on the internet and the whole thing. 2:20:39.7 SC: It doesn’t take that long to be honest, it’s not MPR, as I tell my guests. I don’t put a lot of incidental music in there, I don’t do a lot of editing of cutting and pasting to put the conversation in a better order or anything like that. What we say is 99% of what you hear, roughly speaking, because as I know, as I’ve said, it’s not what I want to mostly do from an hour and hour basis. The… Other than… Sorry, I guess the other thing, the other major aspect here is that for some guests, I really have to catch up a little bit on what they’re doing. I have to read their stuff or listen to them give a talk or something like that, do research, and that’s a different amount. Obviously, if I’m doing John Preskill talking about quantum computing, I know enough, I don’t need to do a lot of research ahead of time, but for other people, I need to do some. 2:21:41.2 SC: That does take time, but that’s part of the reason why I’m doing the podcast, that was part of my self-motivation to nudge me into reading some of these books and learning some of these ideas that I wanted to for a long time. So other than that research aspect, the whole recording, organising, editing, publishing, takes a day, roughly speaking. So one day a week, not including the AMA, which is a whole another thing, but one day a week, I can get the podcast out, which is about the maximum to be honest that I would want to devote to this unless I became a full-time podcaster, but given that there’s other things I wanna do, writing books and doing science, one day a week is a pretty big commitment, actually, but it’s not overwhelming, and that’s why I’m usually doing these AMAs on weekends and so forth. So I think that is almost everything there is to say about how the podcast gets done. 2:22:36.4 SC: Gregory Mendel says, “A priority question, consider that you measure an electron spin along an axis A in St. Louis. Spin up, you go to Los Angeles, spin down, you go to New York. Your spouse subsequently measures the spin of the same electron along axis B and goes to Minneapolis, if the spin is up and to New Orleans if the spin is down.” So if A equals B, I think you and your spouse are in the state, which is a superposition of, I went to LA, my spouse went to Minneapolis, plus I went to New York, my spouse went to New Orleans, because they’re measuring the same spin as I am along the same axis so they’re always gonna get the same answer. It’s just a single electron, not two entangled electrons. 2:23:21.7 SC: Now, the question continues, “Your best friend calls you and observes your location. Your spouse’s best friend calls and observes your spouse’s location. The friends correlate their results and they repeat for axes combinations, yours and your spouses of AB, AC and BC.” So these are different axes along which we could measure the spins. Question, “Is Bell’s inequality,” and then he writes up Bell’s inequality, “Violated as per quantum mechanics proving that you and your spouse were in two places at the same time or not?” So Gregory, I feel bad because I know this is a priority question, but I don’t understand what it is you’re asking. In the original formulation, you said that A was equal to B, but then later in the question, you seemed to be leaving them as independent variables where we have AB and CB different axes. 2:24:13.0 SC: So let me say things that I think are true and hopefully they will provide an answer to your question even if it’s not exactly matching on to the setup that you gave for the question. If you do this kind of experiment, so I presume you’re saying that you’re asking this question in the context of many worlds. If it’s just a Copenhagen question, then quantum mechanics is quantum mechanics, and you’re gonna get the predictions, the quantum mechanics coming out true. Now, the short answer is, in many worlds, quantum mechanics is still quantum mechanics, and you’re going to get the predictions of quantum mechanics, then you will violate Bell’s inequality. Yes, but just to be clear about what happens. If my spouse and I measure electrons along different axes, then in general, there will be four branches to the wave function. 2:25:05.1 SC: So in your original setup, there’s just two branches, one in which I go to LA and she goes to Minneapolis, the other one of which I go to New York and she goes to New Orleans. If they’re not the same axis, there’s gonna be four branches: Me, LA and her either Minneapolis or New Orleans; me, New York and her likewise. And you have to do this many times to actually measure a probability, to even define what you mean by the measured outcome of the probability in axis A, axis B, etcetera. But given that understanding of what actually happens in many worlds, all the ordinary predictions of quantum mechanics are completely true, including the violation of Bell’s inequality, and there’s never true that we were in two places at the same time, we were in two different places on different branches of the wave function. Okay? But that’s just the wonderfulness of many worlds, you can get all of the quantum mechanical predictions out of this situation perfectly well. So, maybe I’m missing something subtle that you’re asking in that question, but I hope that set of true things I just said about many worlds helps answer it. 2:26:15.7 SC: Piu, spelled P-I-U, says, “Priority question. If you could ask an all-knowing entity any three questions about the universe we live in, what would they be?” Again, I’m gonna apologise ’cause it’s a priority question but I don’t have a great answer. I wanna know what the laws of physics are, okay? See, that’s the problem with these all-knowing entity questions, are they a generous charitable entity who would interpret my question in a useful way? If so, I would ask what the actual laws of physics are, and hopefully that would include a knowledge of the ontology of the universe, what the universe is, in addition to how it behaves, the actual laws of physics. Then I might ask a cosmological question about… Well, actually, my second question would be, are there objective moral truths, because I am pretty sure the answer is no, but if the answer is yes, which would surprise me, but I’m open-minded, then my third question would be, well, and what are those? [chuckle] But if not, I would probably default to a third question as something like what happened at the Big Bang or what happened… What is the history of the universe? How long can the answer be? Tell me everything that happened to the history of the universe would be a question you’d be tempted to answer… To ask but then they might go on for too long, so you would want some algorithmically compressed version of that question to be asked. 2:27:40.7 SC: Greg Fife says, “At this time, what is your Bayesian assessment of the possibility that the Fermilab g-2 result actually demonstrates new physics? If it does try out to be new physics, do you have any view on what that will most likely turn out to be?” So remember we talked about the g-2 result in my solo podcast from a few weeks ago, the muon magnetic moment experiment at Fermilab. I think that right now, I’m in a place that says it is less likely to be new physics than not, and that has nothing to do with the actual experiment or the experimenters, and more to do with… We’ve been down this road before. There’s been a lot of claimed anomalies in the standard model, particle physics, none of them have really panned out. 2:28:27.6 SC: Remember, if I’m remembering correctly, and hopefully you also do, the situation with the Fermilab result is that the experiment is pretty clean, like they really have measured the magnetic moment of the electron, of the muon rather. The difficult thing is the theoretical prediction, because in that calculation, there are contributions from strongly interacting particles, quarks and gluons, and those are hard to get right, and it turns out there are two different theoretical predictions for those contributions, one of them, the one that was used in the analysis gives you that discrepancy that they claim between theory and experiment. 2:29:11.8 SC: But there’s another calculation done with lattice QCD, which actually agrees with the experimental result. So if you’re looking for a way that the result might actually not be new physics, it’s pretty obvious what it would be, that the theory prediction is just a little bit off. For what it might could be if it is new physics, I don’t know, I don’t have a favourite example of what it might turn out to be. I think that there’ll be an embarrassment of riches, so I don’t even have a strong candidate for what the most likely thing is to be. 2:29:45.9 SC: The next two questions, I’m grouping together, and they’re kind of related to that one, because they’re about virtual particles and quantum fields. So Cooper says, “Quantum magazine ran an article about the results of a recent muon g-2 experiment, which included this quote, ‘The particle you thought was a bare muon is actually a muon plus a cloud of other things that appear spontaneously.’ I’ve always imagined the quantum foam as something that exists everywhere regardless of the local presence of any real particles. This quote, however, makes it sound like virtual particles only exist in clouds around real particles. Is the quantum foam something that particles travel through or is it something that’s created by the local presence of a particle?” 2:30:27.0 SC: So it’s both is the short answer, the longer but more accurate answer is that the whole picture of the quantum vacuum as consisting of these particles popping in and out of existence is deeply misleading, still a remnant of our insistence on thinking of the world as something visualisable and classical rather than quantum states of fields, which it really is. But to get to the essence of your question, in quantum field theory… Oh, I’m sorry, I forgot to ask the other question, the next question by Peter Waley is, “It’s my understanding that particles like photons are perturbations in the electromagnetic field. My question is, what makes the disturbance in the field? If it’s particles, what came first, the chicken or the field?” Okay, so I’m gonna answer that implicitly, while I’m answering Cooper’s question. 2:31:16.6 SC: The world is made of fields in the quantum field theory limit anyway, gravity is probably a different story, but what we’re talking about here in particle physics, there are fields that you quantise and there’s a quantum state called the vacuum, which is the lowest energy state of your quantum field theory, and particles are excitations into higher energy states of the fields in that quantum field theory. And when you ask, what is the quantum vacuum? I mean, there’s a simple, dumb but correct answer, which is, it’s the lowest energy state of your quantum field theory. But in fact, there’s another way of thinking about the vacuum state, which makes it much more complicated and interesting, which is to think about it in terms of what the fields are doing. 2:32:01.1 SC: So what that really means is, if I probed the field, if I poked at it and said, what happens? Or if I measure the value of the different fields at different points in space and time with different resolutions, etcetera, there’s a very complicated structure to what’s going on even in the vacuum, even in what you and I call empty space, and you can think about… You’re not forced to, but you can think about that structure as arising from virtual particles popping in and out of existence. Again, all of that is just in empty space, but now let’s go to a non-vacuum state, a state with a little bit of an excitation, a particle travelling through the vacuum, there are already all of these virtual particles in a vacuum in this picture, but now there’s also a real particle, and the real particle interacts with the virtual particles. So, you can think about it either way, you can think about it as the real particle is travelling through a pre-existing cloud of virtual particles and interacting with them and being affected by them, or the real particle travelling is just spitting out and then re-absorbing virtual particles and then being affected by those. 2:33:16.6 SC: Those are two perfectly good ways of thinking about it either way. So you will often hear physicists who know the math underlying this, they’ll talk about it either way, depending on what is most convenient for the purposes they’re talking about at the time, and you know, this is something you should always keep in the back of your mind, physicists who do quantum field theory, etcetera, blah, blah, blah, are constantly translating math into words, and the language is never perfect, the translation is never exact. So much of the time, not all the time, but much of the time, when you hear two things that sound a little bit incompatible, it’s just that the words that are being used for the translation are slightly different in the two different cases. 2:34:01.8 SC: Okay, a person who has named themselves anti-de Sitter space is asking a priority question. “In your podcast with Max Tegmark, he proposes the mathematical multiverse in which our universe is just one of many mathematical structures, all of which are equally real. As a non-physicist, I often wonder how ideas like this arise when a physicist proposes an idea like this. Does it originate from some physical intuition, an attempt to be original or something else? What credence should we or can we assign to theories that are not testable?” I think the general question of where new hypotheses and science arise is a good one and a hard one, it’s a combination of psychology and sociology and science. Where does creativity come from? For that matter. 2:34:52.7 SC: I don’t think when you say an attempt to be original, there’s a positive valence to that and a negative valence. You’re not gonna propose a theory that’s not original ’cause someone already proposed it, it’s not that it’s not a good idea, but they’ve already proposed that one. So it’s not that you’re attempting to be original, but you’re attempting to get to the truth, and maybe the ideas that currently exist, maybe none of them is the truth so you wanna be original in the sense that you’re proposing a new idea that has some non-zero chance of being true, that’s a perfectly valid motivation to have when you’re doing these. 2:35:25.5 SC: As to why you pick the specific hypotheses to look at, yeah, physical intuition has to come into it, and again, it goes back to what Lee Smollen and I discussed, and I just talked about right now… And Simon Dale and I discussed, different people bring different intuitions to the table or different commitments to different precious principles, right? One of the great things about Lee Smollen is, he’s more upfront about what his most important principles are. I don’t agree with his principles, but they’re his principles and he’s clear about them, which I think is a good thing to be. In terms of assigning credence to theories that are not testable, I don’t know, I don’t think that’s a very good philosophy of science attitude to have. We should be Bayesians, we should say, “We assign priors to different hypotheses about the universe based on whatever criteria it is that we use to assign priors,” and priors need not be universally shared, you’re entitled to your own priors, but not to your own likelihoods. Different people value different things differently, simplicity, fruitfulness, comprehensiveness, and those will affect their priors. 2:36:38.0 SC: But then we all agree on how the priors are updated when we get new evidence, we try our best to say where the likelihood of that evidence would be under the different theories assigned to the priors, and we move forward from there, but the evidence can take on different forms, sometimes it will be experimental data, sometimes it’ll be new theoretical insights. Sometimes it’ll be realising that oh, this theory is actually more complicated or more vague than I thought. That counts as new evidence that you can use to update your priors. So that’s the way I’d like to think about thinking about evaluating theories rather than just saying this very, very crude idea, this is testable, this is not. It’s more complicated than that. 2:37:24.9 SC: Eli Graham says, “Priority question. It seems to me that quantum physics and the philosophical movement of phenomenology, Husserl, Heidegger, Merleau-Ponty, etcetera, appeared and developed at about the same time in the same intellectual climate. Do you think quantum physics and phenomenology influenced each other?” So when I first read the question, my answer that I was tempted to give was, “Nope, they did not, at least in the direction of phenomenology in this sense of continental philosophy phenomenology influencing quantum mechanics. What influenced quantum mechanics was black body radiation and the spectrum of the hydrogen atom and things like that,” but that would have been bad. I’m glad I didn’t just give that answer because the true answer is, I don’t know, and in fact, it’s perfectly possible that there was influence in either direction, and I just don’t know the history of it well enough, but I did realise… 2:38:22.5 SC: I forget what I’ve said this on the podcast, but I did amusingly… I think I tweeted it. I was, for some weird reason, reading about Daoism and one of the web pages I was reading about, it mentioned that Niels Bohr was knighted or got some sort of honour in Denmark, and as a result, he got to design his own coat of arms, and he decided to in his coat of arms include the yin yang symbol, because he was thinking about complementarity in quantum mechanics, particles versus waves or position versus momenta, two things that cannot be simultaneously described at the same time, but are both different aspects of the same underlying thing. 2:39:10.1 SC: The point of all that is that these people were highly educated. This is back when the centre of Physics was still in Europe, not in the United States, and these people all had philosophy educations, different levels of sophistication, but nevertheless, it would not be at all impossible to imagine that ideas being put forward by the philosophers at the time did seep into the space of possibilities that were being considered to explain the data by physicists. So I don’t know what the answer actually is, but I’m willing to imagine that there was a connection there. 2:39:47.9 SC: Anonymous says, “Are you optimistic or pessimistic that the USA and the world will meet the required greenhouse gas reduction targets in time? If pessimistic, do you sometimes think about whether you as a smart person with a large network and a large audience could do more to steer it in the right direction, e.g. With your choice of guests or the questions you asks, etcetera.” So I’m pessimistic that we’re gonna reach our required greenhouse gas reduction targets. The track record is not encouraging, some governments are into it, some are not, those governments change with times, you can never predict 10 years from now what different countries are gonna be doing. However, then you ask, could I do more to steer it in the right direction? Well, it’s hard. If I really thought that there was something I could do fairly straightforwardly, that I would have a noticeable effect on the direction of the world and climate change, then I would do it, and we’ve had people talking on the podcast about climate change; Naomi Oreskes, Michael Mann, Ram Asnam. 2:40:56.5 SC: But I suspect that in terms of the question, we should do something or we shouldn’t suspect. I’m mostly preaching to the converted here. I suspect that most people, not all, but most people listening to this podcast, appreciate the importance of the dangers of climate change and the importance of doing something about it. It makes me think about authors with Twitter accounts, and I know that I’m one of them, but when an author has a new book out, they can’t help, but on their Twitter account, just talking about their book all the time. And I get it, they’re trying to reach new people and make them buy the book. The problem is, they’re reaching the same people over and over again, they’re not reaching a new set of people with every tweet. 2:41:40.8 SC: Likewise with me in the podcast, I have my audience, but it’s the same audience every time I say this thing. Saying it more and more often reaches a point of diminishing returns very strongly. And honestly, what is it that people can do to help with climate change? I’ve said this before, here in the United States, which is the conditions I know best, the single most effective thing you can do to help fight climate change is get Democrats elected into office. We have strong political polarisation in this country, where if party A takes a position, party B reflexively takes the opposite position, and Democrats roughly speaking, are on the side of trying to do something about climate change and Republicans roughly speaking are not. That’s much more… And any individual person can’t do that much on those directions either, but it’s more than recycling or turning off your light bulbs, or even driving electric car is likely to do. I think that these are problems that require systematic solutions, not valorous individual choices, and the way that the systematic solutions are gonna be implemented is through governments, get a government in place that wants to solve the problem, and then once that’s done, then you can tweak it, even if a government is more… Even if a political party is more devoted than the opposition party. It still doesn’t mean they’re as devoted as they should be, so you can still try to pressure them once they’re in office. 2:43:06.7 SC: Okay, Charles Hurt says, “Priority question. One of the touted benefits of the concept of inflation, cosmological inflation not economic inflation, is that it solves the horizon problem. I know some people like Roger Penrose don’t believe in inflation, and I think that you’ve said in the past, you’re 50-50 on it. Suppose that evidence, empirical or theoretical, is founded clearly to your satisfaction, completely discredits the concept of inflation. Can you speculate on what other kinds of processes or events might then need to be brought into play to solve the horizon problem?” That’s a very good question, and the short answer is no, I don’t have any good second option. And I don’t like the horizon or flatness problem, that’s something maybe I can talk about on the podcast. I have blogged about it before, I don’t… The early universe is in a finely-tuned state compared to our natural expectations of what it might be. That is a statement that I agree with, but how to characterise what the fine-tuning is, I don’t think that either the horizon or the flatness problems are very good ways of doing that as I’ve written about before. 2:44:14.0 SC: But it is fine-tuned and the universe is smooth at early times. That’s really the problem. Why is it smooth? Inflation would make it smooth if it began, and my problem with inflation is I don’t know why it ever began in the first place. But whatever thing caused inflation may be to begin, whether it’s some feature of the wave function of the universe, some quantum gravity thing or some other dynamical mechanism, maybe you could have skipped the step of leading to inflation and just given us a big, smooth universe. When I say big, I mean like a centimetre across. When you take our observable universe and you extrapolate it back in time, it reaches… The expansion rate was the Planck scale, if there’s no inflation. The expansion rate was the Planck scale, or let me say it this way, when the expansion rate was the Planck scale, the size of our currently observable universe was about a centimetre across or a millimetre or something like that. 2:45:11.3 SC: And the problem is, even though that seems small for our whole observable universe, it’s an enormously big size compared to any particle physics length scale, like the Planck scale or the proton mass or whatever. So that’s why I say make a big universe that is smooth. I mean, a centimetre sized universe, that’s one of the wonderful things about inflation is that you could plausibly begin the universe a Planck size across and then inflate it up to be a centimetre across and make our universe that way. But we don’t know if it’s true or not, and I don’t even have any second favourite option. 2:45:44.2 SC: Sandro Stuckey says, “My question is about Mach’s principle. I’ve heard it’s stated as something like the large-scale distribution of mass influences your rotation plane, rotation reference frame or mass out there influences inertia here. These don’t make sense to me. It seems to me that even if there were no stars and planets out there, I could still feel the difference between myself spinning and remaining at rest.” So Sandro, I’m here to help you, ’cause I know the answer to this one. Mach’s principle is wrong. That’s the answer. Mach’s principle… And you’re not at fault for having heard it stated in different ways because Mach never was very clear about it, and other people tried to make up statements that would be a principle that more or less corresponded to his ideas, and they were never completely in agreement, but it was an inspiration for Einstein when he was inventing general relativity. The whole idea that there was a relationship between matter and energy in the universe and space time in the universe. That was the very general idea that helped him go all the way to general relativity, in which the curvature of space time is influenced by matter and energy. 2:46:57.5 SC: But the actual idea that the reference frame that counts as rotating versus non-rotating, for example, is somehow set by matter far away is just not part of general relativity. Once you’re at general relativity, you don’t need Mach’s principle anymore, like you’re intuiting here or implying, there are solutions in general relativity without matter in them at all, and yet there’s still a difference between rotating and not rotating. So forget about Mach’s principle. It was historically interesting. It’s no longer relevant to modern physics. 2:47:30.7 SC: Okay, I have two questions about free will and the block universe. Siddhartha says, “Are libertarian free will and a block universe, mutually exclusive? We can only have libertarian free will if the future is yet to be determined, while in a block universe, the future is already set in stone. Are these ideas compatible?” Red Beard says, “What distinguishes the block universe from someone at the end of time with a knowledge of history? I ask because one has nothing to say about fate or free will, and the other has been interpreted to preclude free will. Is there a philosophical difference between me knowing my history at the end of my life and the block universe view of my life?” So, of course, I’m gonna interpret both of these questions as asking about libertarian free will, which is the idea that we human beings are not completely subject to the laws of physics. Somewhat, we are. Our centres of mass are subject to the laws of physics, but our choices made in our brains are not. 2:48:32.0 SC: And I think this is a silly idea, I think that we are subject to the laws of physics, and in fact, I’m not at all sure what the idea even means. As I’ve said before in other contexts, if a human being was found to be behaving in some way that was clearly in violation of the laws of physics, what I would do is change my idea of what the laws of physics were, and then I would come up with some better laws of physics that included that human behaviour, I wouldn’t just say, “Oh, there are no laws that can explain this,” laws might involve probabilities or whatever, but I’m honestly not sure how to even define what might be meant by libertarian free will in a way that would make it a sensible notion that you could oppose to laws of physics-based universes. 2:49:21.0 SC: Okay, having said all that, I think that most conceptions of free will are mutually exclusive with the block universe. The block universe comes about… Well, okay, it’s gonna depend on what you mean by a block universe. Oh, yeah, it is gonna depend on what you mean by a block universe at all. The original idea of the block universe was in a deterministic context. Okay? So it was really where you thought that you could be Laplace’s demon, know everything about the universe now, and from that extrapolate what would be in the future. So Siddhartha, you said in the… I might not have read this correctly, but you said in the question, the future is already set in probabilistic stone, you can’t say that. Set in the stone means it’s deterministic, you can’t be set in probabilistic stone. So usually, the block universe idea goes hand-in-hand with being truly deterministic. There’s subtleties there, like if you’re a many worlds person, the wave function of the universe evolves deterministically, but the apparent evolution that is actually observed by individual observers is not deterministic. So, you have to finesse it a little bit, it’s not a block universe with a single space time, it’s a block wave function, it’s a single wave function that is determined there. 2:50:44.0 SC: But anyway, if there are laws of physics, probabilistic or not, then there’s not libertarian free will. Libertarian free will is the ability to violate the laws of physics, so those ideas are mutually incompatible. To Red Beard’s question, someone at the end of time with knowledge of history, I’m not quite sure what the relevance of a person at the end of time with knowledge has anything to do with it. The idea of the block universe really, it matters that the future of the block is in some sense inherent, implicit in what is going on right now. So the block universe is more than just the idea that you wait infinitely long and then you get a universe… Okay, everyone agrees with that. Even if you’re not a block universe person, if you’re a presentist, you can believe that you wait until the end of time, if that’s a sensible concept, and then you say what happened. You write a history book. 2:51:42.8 SC: The block universe idea is really, like I said, it goes hand-in-hand with the idea that the future is implicit in the present, and in that case, you don’t need to know about what the future is going to be, and it’s a case that is distinguished from cases where either you have indeterministic laws of physics or you don’t have the laws of physics at all because you have libertarian free will. In those cases, the future is not fixed in terms of the present, and so the block universe idea doesn’t quite fit as well, even if a person at the end of time would have all the knowledge of it. Okay, I’m not quite sure I did a very good job explaining those things, but hopefully the essence came through. 2:52:25.2 SC: Alas Por Yorick says, “The connections between physics and philosophy are fascinating yet it feels to me fairly crucial to a scientific temperament… Sorry, and it feels to me fairly crucial to a scientific temperament, yet my physicist colleagues seem to relish a wholesale rejection of the entire broad tradition of philosophy. I’m often trying to get these friends to take philosophy beyond the vague impression of Karl Popper and politics other than naive technocracy seriously. I was wondering if you had any thoughts on where this antagonism comes from and how to counter it as it seems quite pervasive and popular among famous scientists who should know better.” 2:53:04.3 SC: Yeah, I think this is a good question, especially because it doesn’t seem… So number one, I agree that there is, broadly speaking, some hostility in physics circles to philosophy, that’s a very overly simplistic statement, but there’s some truth to it. 2:53:21.4 SC: But it doesn’t seem to be constant over time. As I just said, 100 years ago, the people who you would identify as the leading physicists of their generation were deeply interested in philosophy and philosophy questions, and they would have thought it bizarre to think otherwise. Certainly in the past, or if you go to Boltzmann or Maxwell much less Newton and Galileo and Laplace, these people could barely see the difference between physics and philosophy. So it’s a somewhat recent phenomenon, and it’s not entirely wrong to co-locate the phenomenon with the transition of the centre of the world of physics from Europe to the United States. So, there is a cheap, but may be somewhat relevant fact that Americans have a different attitude about these things than Europeans did. Americans are more direct, pragmatic, gung-ho, get it done, don’t ask too many questions, and that is an attitude that actually fits very comfortably into modern physics. Which is kind of ironic, the idea, ’cause physicists think about weird things. 2:54:35.6 SC: Wave functions of the universe, super strings, the multiverse, whatever, but given that subject matter, physicists tend to be pretty down to earth about it, they wanna know what is the pay-off? If you’re familiar with pragmatism, this American philosophical tradition of pragmatism, which not exactly accurately, but to give you some of the flavour of it, is sometimes acquainted with the notion of the cash value of ideas, the pragmatist vision of truth of an idea was what usefulness does it have in the bigger picture purpose that I have? So if your bigger picture purpose is predicting the outcomes of physics experiments and your view of truth is essentially pragmatic, what is the cash value of these ideas, then you can see how you would be pointed in the direction of ignoring philosophical questions to the extent that those philosophical questions didn’t help you predict the outcomes of physics experiments. 2:55:39.2 SC: Now, I don’t know if this is actually historically accurate, I don’t know how valid this is as an explanation for the hostility of modern physics to philosophy, but maybe it has a part to do with it. How to counter it? Part of it is, there’s a whole another thing going on, which goes back to our discussion of intellectualism, which is that there is a difference between being devoted to a particular field of intellectual inquiry and being devoted to intellectual inquiry for its own sake, just like there’s a difference in politics or in society between… There’s a difference between being devoted to making sure that your group gets the rights and privileges that it should be afforded versus being devoted to the idea that all groups should get the rights and privileges that they should be afforded. And in the case of intellectual effort, again, it’s perfectly valid either way. You can get a lot of progress just by being narrow-minded and sticking, or at least narrowly-focused, let me put it that way, and sticking to the interest of your subdiscipline. 2:56:49.0 SC: And so the other thing to keep in mind is not America versus Europe and physics and philosophy, but just the hyper-specialisation of the academy as knowledge has grown to be influential and productive as a knowledge producer requires increasing specialisation. And so being appreciative of the perspectives and values and techniques of other fields is generally becoming devalued, and it’s not just physics and philosophy, that you’d pick out other sub-fields just as well. And I say this, and by the way, you should take everything I say in these discussions with a grain of salt ’cause I have a dog in the fight. I am very, very devoted to interdisciplinary discussions. I am very, very frustrated by the fact that the rest of the academy does not share my interest in these things all the time. Many administrators or whatever, talk a good game when it comes to interdisciplinary engagement, but the way that we do the organisation of universities in the modern world is department by department, and departments are self-perpetuating. The hiring and promotion of new professors is at the 98% level done by the professors in a department that already exists. 2:58:18.6 SC: So you can say, oh, wouldn’t it be nice to hire someone who crossed over departments? But it’s really hard to do in practice, and this leads to the increasing siloisation of the academy, and so that’s a whole another thing that probably feeds into this. But, look, everything I’m just saying right now is vague anecdotal speculation. These kinds of questions are hard and to do them seriously, you have to do some real work in, I don’t know, history, sociology of academia, and that is not my area of expertise, I’m afraid. Sorry, the other part of the question was how to persuade them to think otherwise. There’s two angles to take. Point zero is, is it worth trying to persuade them that philosophy is interesting or useful? Just like people on the other side of a political divide or any other highly polarised divide, you first have to decide whether the person you’re talking to is open-minded. 2:59:22.9 SC: If they’re just devoted to their position, then don’t waste your time, but I think there are a lot of people who are dismissive of philosophy or other areas of intellectual inquiry more generally who just don’t know anything about them, but might be open-minded if they did. So then the two points I wanted to make about that are number one, point out that there are some tiny parts of physics where philosophy is important, but point two, most of physics and most of philosophy are separate, but nevertheless valid. So in other words, it is simultaneously true that if you’re doing something like thinking about the measurement problem in quantum mechanics, thinking about the role of probability in cosmology, thinking about the origin of the arrow of time, these are very, very valid, obviously interesting physics problems for which philosophy has a lot to say, and philosophy really is useful. 3:00:21.2 SC: At the same time, there are plenty of physics problems for which philosophy is not useful, that’s just obvious, and so there’s no like no necessary connection between physics and philosophy. If you wanna calculate a Feynman diagram, you don’t call a philosopher. And at the same time, there’s a lot of philosophy questions that have no impact on physics; morality, ethics, aesthetics, right? They’re intrinsically interesting in their own right, so even though I want to stand up for the usefulness of philosophy to certain circumscribed parts of physics, that’s not why I think philosophy is interesting; philosophy is interesting because philosophy is interesting, just like physics is interesting because physics is interesting. That’s the attitude that I would mostly push on, but there are other people who are gonna be more persuaded by the fact that, oh, if you want to understand that the nature of entropy and the arrow of time, you better talk to some philosophers ’cause they had been more clear-headed about those questions than physicists have traditionally and recently. 3:01:19.8 SC: Frederick Cloch Lauritz says, “I’ve heard you mentioned that many worlds doesn’t have any philosophical implications.” I think what you mean is moral or ethical implications, but I get it. “How about a case where a person is given the choice to sacrifice themselves to save humanity, if the person believes that this universe is all there is, they would be inclined to sacrifice themselves because the alternative is the end of humanity in a final and absolute sense. But if they knew that there was a near infinite amount of other worlds where humanity would continue on, then the person wouldn’t have to worry as much about destroying humanity because this is just one world out of many.” Well, so I think it doesn’t work. There is, as I said, in something deeply hidden elsewhere, there is a version of morality in which many worlds does matter. It’s basically sort of a non-linear version of morality. If you don’t care about the average value of things, but you put in an absolute value on things, like if you put an absolute value just on if you were a utilitarian, for example… Sorry, utilitarianism doesn’t work. What works? 3:02:26.3 SC: If you think that there are some things that could happen that would be sort of absolute evils, so evil that it didn’t matter if no matter how many other good things happened. So it was the opposite of being utilitarian. If there are certain things that you never, ever, ever, ever want to occur, then you would try in the many worlds interpretation, not just to make them unlikely, but to make them sort of impossible in any world. Am I getting that right? I don’t think I’m saying this very, very correctly, but the point is, I can imagine cooking up very sort of artificial-sounding versions of moral codes that would in fact be influenced by many worlds. Oh, yes, I remember. Okay, yes. Sorry, I’m getting the example wrong. Here’s the example that I came up with in something deeply hidden. Imagine that your highest value was equality. In other words, you don’t care how rich people are, and we’re just gonna use money as a proxy for value in this case. You don’t care how rich people are, what you care about is that they’re all equally rich, right? And then someone comes up to you and proposes a thought experiment. No, proposes a real experiment. 3:03:39.9 SC: They are a multi-trillionaire; very, very wealthy, and they say, “We’ll measure a spin of a particle, and if it spin up, I will give every human being a dollar; and if it spin down, I will give every human being $2. Okay, now, in the ordinary quantum mechanics, you would say, “Sure, take that. I would let you do that. I get either a dollar or $2, and so does everyone else. Everyone benefits.” If you were a utilitarian and you believe in many worlds, then you would also say, “Yeah, sure. In one world everyone gets a dollar, and in the other world, everyone gets $2, all good.” But if for some reason, you had this cockamamie moral theory that says that inequality is intrinsically bad even across the multiverse, then you would say it’s bad that in some part of the multiverse people get $1 and in the other part they get $2, and therefore I will not let you do this experiment at all and everyone gets zero dollars that keeps them equal, right? 3:04:43.0 SC: Okay, that’s a very, very silly moral code, I think, but it is an example of how in principle, many worlds could matter. Now, I don’t think that works for the example that you’re giving, in this sort of given the choice to sacrifice themselves to save humanity. You didn’t phrase the way that the thought experiment and the way the quantum mechanics has anything to do with it. So if you’re just given… If someone holds a gun to your head and says, “I have a trigger that will destroy the universe, and either you let me kill you or I do the trigger, and everyone dies.” By the way, if everyone dies, why not just sacrifice yourself? I don’t see… You’re not getting advantage out of this, for not sacrificing your selves, but okay, let’s say that you were sort of cowardly and didn’t wanna do that, there’s no quantum in there, there’s no branches of the wave function in there, there’s one branch and in that branch, you will either save the humanity or not. 3:05:45.4 SC: So what you need to imagine, is that there’s some probability that… Some quantum probability, that the universe is sacrificed. So, would you sacrifice yourself to give the population of the Earth a 50% chance of dying or being saved, versus not sacrificing… Sorry. Yeah, I think that the right way to do it is, if you sacrifice yourself, there’s a 100% chance that no one dies, if you don’t sacrifice yourself, there’s a 50% chance that everyone dies and a 50% chance they don’t, because then, you can actually have something to weigh against the different occurrences in different branches of the wave function. 3:06:32.0 SC: But in that case, I would argue, just as I argued in the book, it’s… The analysis is exactly like a single world in which there’s just a probability of it. So for me, and maybe not for other people, depending on your moral code, but for me, there is no difference between a scenario in which there’s a branch of the wave function in which everyone lives and an equally sizeable branch in which everyone dies, and a 50-50 chance in a single world, of everyone living and anyone dying. So that’s a vivid difference conceptually, but I think it doesn’t make any difference to me actually, in the actions I would take to save the world, because for exactly the reason that if you’re going to be… If you’re gonna believe in many worlds, then you believe that for people living in that universe, their experiences are exactly the same as people who live in a single universe, where these things happen with a certain probability. 3:07:37.9 SC: So it’s very much like the quantum suicide experiment, where you say, “Well, if I die in a certain branch, I don’t know, I only experience life in the branches where I’m alive.” That’s not valid analysis, in my mind, because as we said before, in a different context, right now, before the experiment is carried out, I care whether I’m gonna survive or not. If there are a classical world, and someone says, “Would you mind if you were killed instantly and painlessly?” I’d say yes. And the fact that I won’t be around to regret it, is completely beside the point, I would regret right now the prospect of no longer existing. And that’s just as true in the many worlds interpretation of quantum mechanics. 3:08:23.5 SC: Now, others might disagree. So again, this is stuff where it requires thought, because the metaphysics of Everettian quantum mechanics is very different than our usual way of thinking. So by all means, think hard about this. I’m not gonna say that I’ve given you the final answer, but that is how I think about it. 3:08:43.8 SC: Hilbert Spaceman says, “What would constitute experimental evidence, that Schrödinger’s equation does not always hold? For example, why isn’t every measurement of a spin evidence of this?” 3:08:53.9 SC: Well, the answer why every spin measurement is not evidence of that, is that in many worlds, Schrödinger’ equation does always hold and yet it predicts exactly what we see in spin measurements. So clearly, by itself, that is not evidence. What is evidence for, is that in our branch of the wave function, we cannot explain what happens by assuming the… That we see the whole result of the Schrödinger equation holding, we’re only seeing part of the result. But what you would need, for evidence that Schrödinger’s equation doesn’t hold, is to see wave functions violate the Schrödinger equation without being observed, without being measured, without being decohered, without becoming entangled with the rest of the world. When we in the real world, do a measurement, we entangle things with the rest of the world. You can do experiments where you wait for things to violate the Schrödinger equation, even though they’re not being observed, and if a wave function spontaneously localises, that would have experimental impacts. Energy would not be conserved in a very obvious way, for example. So you need to be careful that you’re not secretly disturbing the system, but that’s always gonna be the case, when you’re trying to do at a careful physics experiment, you need to be careful. 3:10:09.4 SC: Fred Alexander says, “Would you consider allowing one of your Patreon members to interview you for a Mindscape episode?” 3:10:19.3 SC: I would consider it. I’m not sure what I… What makes me pause about that, is not that it’s not an interesting idea, but I’m not sure if it distinguishes itself from what we’re doing right now, with the AMA’s. In the AMA’s, in some sense, you’re getting an interview of me, from 100 different Patreon members, each asking one question each. So I’m not sure there’s that added value added to that, plus there would be set up involved. But again, like many other things, I have enormous appreciation for the Patreon supporters here. If there is a huge well of support for an idea like that, I would take it seriously. Let’s put it that way. 3:11:04.6 SC: DMI says, “Doesn’t the Reeh-Schlieder theorem resolve the black hole information paradox?” And there’s a longer… If you wanna go to the website and go to Patreon, look at the question being asked, there’s a much longer explanation of what he means by that. 3:11:20.4 SC: The Reeh-Schlieder theorem is basically just the idea that… Well, it’s stated in a particularly fantastical sounding way, but what it comes down to, is the fact that in a quantum field theory, every little bit of a quantum field in one location of space shares some amount of entanglement, even if it’s a very tiny amount, with a bit of the quantum field in the vacuum state, this is all for the vacuum state, at any other point in space. So you’d pick two points in space, take a small region around them, a cubic centimetre or whatever, there are fields inside them, and it is a feature of the quantum vacuum state, that they are entangled. Maybe only a little bit of entanglement, but there’s some entanglement between those quantum fields. And as a result of that, the Reeh-Schlieder theorem says, if I project onto the field in one region of space in exactly the right way, I can turn the rest of the wave function of the universe into essentially whatever I want. 3:12:21.8 SC: Now, that’s a little bit misleading, because of the verbs in there don’t actually correspond to things you’re actually allowed to do. So if I have to make things simpler, just take a single particle, which is spin up plus spin down, an electron that is in a superposition of spin up and spin down, and I measure the spin. What happens when I measure the spin is, there is a probability I will see it spin up, and a probability I will see it spin down, and after, I can describe that by saying, “I projected onto the spin-up part or the spin down part.” The problem is that you can’t, ahead of time, decide which thing to do. In other words, if your electron is in a superposition of spin up and spin down, you can’t say ahead of time, “I’m gonna measure it and get spin up.” If you could, then you could do superluminal communication between Alice and Bob, having entangled particles. I think this is the thing that people implicitly suspect you can do, when they think that entanglement leads to superluminal communication. 3:13:27.2 SC: If Alice and Bob have two particles and they’re entangled in opposite directions, so they’re anti-parallel, then the correct thing to say is, Alice measures her spin, it’s either up or down, Bob measures his spin, it’s either up or down, they’re correlated, if one is up, the other’s down, but they don’t know, neither one of them know the answer that the other one got. If Alice was able to measure her spin, in a way that guaranteed that she could get spin up, then Bob could measure his spin and he would always get spin down, and if you get that with 100 spins, he would go, “Aha, they’re all spin down, Alice is sending me a message.” But Alice can’t do that. So, typically, they’ll get 50-50 on both of them. 3:14:10.4 SC: So likewise, in the Reeh-Schlieder theorem, there is something that can happen in the cubic centimetre of space that you look at, that in principle, if that thing happened, would cause the rest of the state of the universe to change in an incredibly dramatic way. The example that’s often given is, you could make the Taj Mahal appear on the moon, but you can’t force it to happen, you can’t choose to make that happen. There is a thing that could make that happen, but it’s incredibly extraordinarily super duper unlikely. That’s why these things don’t ever happen in the real world. 3:14:46.5 SC: So, the question about resolving the black hole information paradox, if… I tried, DMI, to understand your longer version of the question, but maybe like everyone else, I’m misunderstanding it. The Reeh-Schlieder theorem implies that there is entanglement between degrees of freedom inside the black hole, and degrees of freedom outside the black hole. That’s true, but that doesn’t help you get information out. That’s the point. The point is, you wanna get the information about the quantum state of systems, out of the black hole, and if you just believe the most naive version of what happens in space time, the part of the quantum state that is inside the black hole hits the singularity and disappears, and what you’re left with outside is a mixed quantum state that has just had the information about what fell in, erased. It’s the equivalent of Alice falls into the black hole, Bob has a spin, and after Alice falls in and the black hole evaporates, Bob’s spin is 50% up and 50% down, but the exact entanglement with Alice’s spin is just lost forever. So that’s the information loss puzzle. 3:16:01.1 SC: So Reeh-Schlieder relies on the fact that there’s entanglement, but that’s not nearly enough to help you get the information out, roughly speaking. 3:16:09.7 SC: Rebecca Lashua says, “On the internal inflation Wikipedia page, it says, the inflationary phase of the universe’s expansion lasts forever throughout most of the universe. Does that mean, at some finite distance from the Earth, there’s a patch of space time undergoing exponentially fast expansion, or are the parts of the universe undergoing inflation, disconnected from our universe in some way?” 3:16:32.8 SC: Yeah, usually, eternal inflation is not perfectly well understood, so there’s details and different versions of the scenario, but roughly speaking, the idea is that, yes, some finite distance away from the earth, there should be a patch of space time undergoing exponentially fast expansion. In fact, in most versions of this, most of the volume of the universe is undergoing exponentially fast expansion in eternal inflation, if eternal inflation is happening at all. They are usually outside our causal horizon, so it’s not that they’re disconnected physically, but we can never get there. We could never get there even if we start at the speed of light and moved right now. 3:17:15.0 SC: Gillis 15 says, “Although the events were detected by LIGO and Virgo in 2020, a new paper just released confirms for the first time, a black hole neutron star merger. Will the black hole will be able to tear apart the neutron star? Is the star spiralled in, or will the neutron star cross the event horizon whole?” 3:17:34.2 SC: So this is an interesting astrophysical question, and I’m addressing it not because I know the answer, not… This is a little bit too down to earth for me, but I think that part of the excitement of my impression from reading the popular articles about this particular event, is that we learned something about this particular question. It’s complicated. Neutron stars are messy, black holes are non-linear, and there’s a lot going on, and Einstein’s equations are hard. So you might guess that it’s plausible either way, that the neutron star is mostly swallowed whole or that it is gradually torn apart. If you are really good at doing simulations, you should be able to make a prediction ahead of time, but I don’t know what the state of the art of those simulations is. 3:18:25.5 SC: It seems, from what I understand, the fact that we didn’t see an electromagnetic counterpart of these particular events, implies that, to a very good approximation, the neutron star was swallowed whole. And if you see the little pictures, the little animations that they made, that’s basically what happens. The neutron star comes very close to the black hole and then disappears very quickly. So I think that’s what happens, but I don’t know if that’s always what’s gonna happen, or if this was particular to the cases of these particular in spirals. Again, I’m not enough of an expert to know. 3:19:01.6 SC: Naive Bayesian says, “How would a good Bayesian reconcile the following hypotheses? Number one, reality is real. Number two, reality appears real because Descartes’ demon is tricking you. Three, reality appears real because you’re a Boltzmann brain. Four, reality appears real because you’re in a brain simulation. Five, reality appears real because of literally any other ad hoc tom foolery example.” 3:19:27.7 SC: So you can’t. You can’t really reconcile those, but you have to deal with them in some way. So I think that there is a strategy that more or less works, but I’m not gonna pretend that this is a situation where all of the I’s are dotted and the T’s are crossed, that I understand exactly what’s going on, or at least have a good set of advice to give you. So for those of you who are not familiar with all these different phrases here, this is the idea that either the reality you see around us is basically real. There really is a desk in front of me, a microphone, a computer, and so forth, or that somehow, we’re in what we call a sceptical scenario. Somehow, that there is a kind of reality that is very different than the apparent reality around us, but you are being fed an illusion. So this is different and there’s a subtle difference, I wanna emphasise the difference between something like the difference between the manifest image of the world and the scientific image of the world. 3:20:32.0 SC: So the fact that there’s a table in front of me and a microphone, etcetera, none of this, none of these words, table, microphone, appear in the standard model of particle physics. The whole classical world I see around me, is a very tiny slice of the real world, if you believe in the many worlds interpretation of quantum mechanics. So those are both senses in which there’s a difference between reality and what you see. But, in both cases, that’s just because you’re seeing a small part of reality and you’re still seeing part of reality, what you’re seeing is part of reality, even though it’s not all of it. That’s not what’s being talked about here, what’s being talked about here, is the idea that there’s reality and there’s what you see, and they’re very different because you’re being misled or tricked in some way. And so it’s not that hard. Going back to Descartes, who is a master of this, it’s not that hard to imagine sceptical scenarios that are so convincing, that no experiment you could do, would reveal that what you were experiencing was not reality. 3:21:34.4 SC: Maybe a very good demon is tricking you, or you’re in a simulation, or brain in a vat or whatever. I think that’s possible. I think that is something that is on the table. Maybe we are being tricked. But as a good Bayesian, and this is the question you’re asking, what does a good Bayesian do, in this case. You have to discount it. I think a good Bayesian says, “Look, if I can take what I do observe and explain it in a variety of different ways, and some of those are simple and coherent and self-contained, and others are awkward and artificial and complicated and weird, I’m gonna pick the simple ones.” There’s just nothing added by imagining that we are a Boltzmann brain or a simulation or being tricked by an evil demon. So I think that’s the way to go, in the real world. You wanna make sense of the world. And so, the way that I talked about it in my paper on why Boltzmann brains are bad, is that, to exclude the Boltzmann brain possibility by just setting the prior credence on it to a very, very, very low number, keeps us in a cognitively stable situation. 3:22:48.9 SC: If you say, “I’m not a Boltzmann brain,” then you can make sense of the world. If you say, “Well, maybe I am a Boltzmann brain,” then you can’t make sense of the world ’cause you have no reason to trust any of your experiences of the world. So it’s not a useful hypothesis on which to put a lot of credence. That would be my argument. Now, having said that, the little niggling problem that should worry you is, as a good Bayesian, you should never set your credences in any physically possible scenario, to exactly zero, because if you did, then no future evidence would enable you to find that hypothesis, you’ve set it to zero, and you multiply by zero every time you update. 3:23:35.6 SC: So I do think good Bayesians should keep some non-zero credence, even for these skeptical hypotheses. What if the demon reveals themselves and says, “Ha, I was the demon who was tricking you.”? Then you wanna be able to update and go, “Ah, a demon was tricking me all along.” So at the same time, I think you want to set these credences to very tiny numbers, but not to zero. And how do you do that? What do you mean, very tiny? Do you mean one in a million, one in 10 to the million? I don’t know, I don’t know what those things are. 3:24:08.1 SC: Fortunately, I don’t think there’s a lot of credence for any of these scenarios, so I don’t need to worry about it that much, but I appreciate the importance of this kind of question in principle. 3:24:18.8 SC: Fabian Rosedallan says, “Regarding vegetarianism, you’re the only person I’ve heard making a reasonable argument for the justification of eating animals, namely their lack of ability to anticipate their future, their mortality, and so on. The philosopher who got me convinced to veganism, morally put forth the name the trait argumentation style of reasoning about these issues. Namely that if this trait… If a certain trait or lack thereof, is sufficient for killing or eating another being, it should be so, regardless of the species of that being. Example, it’s okay to eat animals because they’re not as intelligent as humans, should then also conclude in, therefore, it is also okay to eat humans with equal intelligence to an animal. Do you agree with this reasoning? If or if not, why?” 3:25:06.1 SC: So I don’t really agree with this reasoning, but let me give a little stage setting here, first. I am a moral constructivist. I think that what we call morality is sort of a systematisation and codification of our moral inclinations, of our deep seated feelings, that certain things feel right, certain things feel wrong, plus, our rationality, the idea that we would like to take these feelings and systematise them into a set of consistent principles. And sometimes, our inclinations might be inconsistent, incoherent, and we have to do some work like, “What do I really care about?” And we might also wanna talk to other people and get more insight into possible ways to be good and bad, and that would also change our minds. So I don’t think that there is a simple and direct road from our moral feelings to morality, as in moral philosophy, but that is the starting point. And because of that, morality is not objective or cut and dried or rigorous or universal. Different people might come up with different kinds of morality. 3:26:16.8 SC: So, all of that is to say, that the answers to these kinds of questions might be different from person to person, rather than me giving you the once and for all final answer, even in my own mind. Having said that… So in my mind, when we come up with rules like, “You shouldn’t eat humans,” or whatever the rules are, you’re coming up with rules, so you’re inventing rules to fit the inclinations you have, and to me, it makes 100% perfect sense to apply those rules on a species by species basis, rather than on an individual by individual basis. You could decide to apply them on an individual by individual basis, that’s a different set of morality, and then you and I could sit down and talk about the implications of that choice, but it’s not my choice, and I don’t think it’s the choice that most people would have. Part of this is why, even though I’m not vegan or vegetarian myself, I have 100% respect for people who are. As a moral constructivist, it would be very… It would be a little bit hypocritical to me, if I said, “Well, you’re just wrong. You vegans.” If you think it’s wrong to kill animals and eat them, I get it. 3:27:30.0 SC: You’re totally valid in that. If you think it’s wrong that I kill animals and eat them, you know what, I even get that, because I’m not a moral relativist, I don’t think that everyone or every society gets to make up their morality, I think it’s perfectly valid to do your best to come up with moral rules and then apply them to other people. I think that… ’cause that’s perfectly… Even if they’re not gonna come up with the same rules, the rules that you come up with, are what you think is moral and not moral. So it’s okay if you judge me badly, I get that. I’m gonna try to… Well, both, of course, think through my own moral reasoning and then also sit down and reason together about what our collective rules should be, about whether eating animals should be legal or illegal and so forth, and hopefully, we can have reasonable conversations about that. 3:28:26.0 SC: But I think that this particular issue of… This assumption that, or maybe not assumption, but claim, that it is somehow incoherent or inconsistent to apply rules to species rather than individuals is completely arbitrary, and ad hoc. I’m applying my rules, I’m telling you what my rules are, I apply them species by species, and that seems perfectly sensible as a thing for me to do. 3:28:53.8 SC: Okay, I’m grouping… We’re reaching the end here, but I’m grouping together a couple of questions. This is about faster than light travel. Somosha Fader says, “The two largest and most popular science fiction franchises involving space travel, Star Trek and Star Wars, share as part of their basic premise, the common science fiction assumption that their methods of near light speed and faster than light travel somehow exempt them from relativistic time dilation. So that got me worrying… Wondering about warm holes. If they could be made traversable and you could use one to cross the galaxy and return, would the temporal effect of relativity still apply, even though you had never moved at relativistic speeds?” 3:29:33.2 SC: And then Damon Winston Bird says, “I have to go to Alpha Centauri and I don’t wanna spend years getting there. I hop in my Alcubierre drive, which is the kind of warp drive solution to the equations of general relativity invented by Miguel Alcubierre, and get there in only one year. After taking care of business there, I hop into my faster than light Alcubierre drive and I’m home again after another year of travel time. Did two years pass on earth as I traveled?” 3:30:00.0 SC: So these are slightly different scenarios. Wormholes versus warp drive. And the worm hole case is actually a little bit more straightforward, but the… More visualisable or conceptualisable, let me say it that way, but the answers are the same in both cases. In both cases, you’re effectively moving faster than the speed of light. Why do I say effectively? Because locally, you’re not. In both the warp drive case, the Alcubierre warp drive case, and in the worm hole case, what you’re really doing is stretching space time, warping space time in such a way… Actually, interestingly, what you’re doing in the Alcubierre warp drive is, literally warping space time around you, even though the idea far post dates the phrase warp drive. 3:30:48.2 SC: So I wonder… I have no idea why warp drive is called warp drive. I think, just ’cause it sounds cool and fast. But the idea that you get it by actually warping space time, is probably not what they had in mind, but it actually fits very well. Anyway, in both cases, you’re looking at a space time curvature set up, a geometry of space time, that is different than just flat space time that you get in special relativity, and this lets you… And you’ve done it in a specific way, that you can compare two different ways of travelling between two points in space. 3:31:26.3 SC: One way is by just going in what you might think of as a conventional path, not being in the warp drive, not being in the worm hole, and then it takes you a certain number of years to get where you’re going, a certain or years to get back, from the point of view of the people who stayed home, or you can get there to the same point, by travelling through this tunnel in space time, either in the warp drive or in the warm hole. So then the question is… Forget about time dilation, that’s not really quite the right way to think about it. The question is, “How much time lapses in your reference frame, according to your clock, as you’re travelling between this point and this other point, either in the worm hole or in the warp drive?” 3:32:08.3 SC: And the answer is, I don’t know. The answer is, it depends on the details of the worm hole or the warp drive. So it is completely plausible, that they’re synchronised in such a way, plausible might be an exaggeration, conceivable is a better word, maybe. So the warp drive is a little bit fuzzier, again, but the worm hole is a pretty direct thing. There’s a distance you travel through the worm hole, and there’s a distance you travel back, and there’s also a time that it takes to go in the worm hole and a time it takes to come back, and I could easily imagine setting up my completely fictitious worm hole system, such that the time that elapsed on my watch, as I travel through the warm hole and travel back, is the same amount of time as elapsed to the people back home. And presumably, I can do the same thing with the warp drive, although I haven’t actually gone through the equations in that case. 3:33:04.0 SC: So in other words, I think it is completely conceivable to set up a system of effective faster than light travel, that works like Star Trek and Star Wars do, that actually in the back of everyone’s minds, they haven’t quite graduated from Newtonian absolute space time to Relativity, and they think that there is something called the simultaneous things going on on Alderaan and Tatooine or whatever, and that’s the narrative frame of reference of the movie. You could imagine something like that. It’s not what would naturally pop out of a realistic space time geometry, but you could imagine it. So if you wanna make movies, go ahead. All of these are really cowboy movies, at the end of the day. When you say you’re in a Starship, really, you’re in a covered wagon going far away, and that’s how they really think about things. This is the right way to conceptualise why they had the adventures that they do. 3:34:04.9 SC: Okay, Anonymous asks, “In episode number 62 with Kate Jeffrey, you both talked about how physicists are essentially certain that the universe won’t last forever, and about how some of the processes that will cause the universe to wind down. You also talked in general, about how ironic it is, when knowledge about physics stays stuck in ivory towers, when the explicit purpose of studying physics is to discover new knowledge for society. I’m curious about how the knowledge that the universe will definitely permanently end has impacted society and culture, or whether that’s an example of something the public really ought to know more about?” 3:34:41.6 SC: So just to be super clear, the sense in which I think it’s pretty clear, it’s never definitive, never 100%. So I don’t wanna say that. But it’s overwhelmingly likely that the universe will end. It’s not that time will end, it’s that the ability of the universe to increase in entropy, will end. There’s a sort of limit about what we can do, as we travel from low entropy to high entropy, there’s only so far we can go, as you can illustrate, by the way, in a Mindscape T-shirt. We have a T-shirt showing the curve of entropy versus time, as well as complexity versus time. It’s not the most physically accurate graph, I just drew it in a little program, but it gets the idea across. 3:35:25.2 SC: Anyway, so it’s not that time will end, but the arrow of time will end, we’ll use up the fuel that we have to keep the universe going. That’s what I was talking about with Kate, in that episode, and with another Kate, with Katie Mack, we talked about different physical ways the actual universe might end, which is a slightly different concept. But it’s an interesting question you ask, about whether or not this idea of the heat death of the universe, that’s really what Kate Jeffrey and I were talking about, the running out of entropy. Does that have an impact on society? Well, because it did, when it came about, when people first started talking about this in the late 19th century or early 20th century, it did leak into the popular imagination. 3:36:13.6 SC: And I’m gonna get it wrong, I wanna say Henry Adams, talked about it. And other people who are not physicists, they caught on to this. Arguably, Nietzsche, in his idea of eternal recurrence, was at least indirectly affected by this discussion going on in physics, before they knew about the Big Bang and things like that. And Thomas Pynchon, more recently, has absolutely written about related concepts. But okay, it’s still not there in the everyday life of popular culture, we don’t talk about it too much, and maybe it’s just ’cause people don’t care. We’re talking a very, very long time scale, and Zach Wienersmith, the cartoonist, was talking on Twitter just today, ’cause he’s writing a book on future space travel, and he says, one of the things that is frustrating about talking to the real gung-ho space geeks, is that the time scales of interest to ordinary people and to the space geeks are very different. So when Zach will say, “Look, I’ll just talk about terraforming planets, it’s just unrealistic. We shouldn’t worry about that. This is not on the horizon.” And the space geeks are like, “No, no, it’s completely plausible. We’ll absolutely do it.” Implicitly, Zach is thinking, within the next century or two, and the space geeks were thinking within the next million years or two. 3:37:43.0 SC: And so, that difference in implicit time scale matters a lot. Real people have difficulty thinking even 100 years in the future. So when you say that the sun will burn out in five billion years and the last star will burn out in a quadrillion years, and the last black hole will evaporate a googol years, their lives are not affected by that very much, they are amused for a minute and they go back to their everyday lives. And you know what, they’re not wrong. I think it’s an interesting thing to think about, but I’m not really sure that it has a huge impact on how we should live our lives here on Earth. 3:38:18.3 SC: Lineo Misiara says, “What would physics look like today if Feynman hadn’t invented his diagrams?” 3:38:26.0 SC: Well, there’s two answers to that. One is, it would look the same, because someone else would have invented them. Feynman diagrams are great, but like most other inventions in theoretical physics, someone gets there first, but other people would have gotten there eventually. So I think it’s overwhelmingly likely that someone would have invented diagrams, Feynman diagrams. They’re just too juicy, too useful, too helpful, and physicists like drawing diagrams. So someone would have invented them eventually, maybe we’d be calling them Weinberg diagrams, or a tuft diagrams or something like that, Whitten diagrams. 3:39:05.8 SC: But the other answer, the other version of the answer is, and maybe this is the implicit thing, what if nobody had invented Feynman diagrams. I think that somebody would have. But what if they hadn’t? Julian Schwinger, who shared the Nobel Prize with Feynman and Tomonaga, basically did the same calculations that Feynman did, without the diagrams. And famously, the joke was that, because they were kind of rivals, Schwinger refused to use Feynman diagrams for the rest of his career. I think that’s not true. I think he did actually use diagrams, but he had a way of doing the same calculations purely algebraically, purely with equations. 3:39:48.7 SC: So, you can imagine a world in which we did the same calculations, but it took longer, they were harder to do because there were a lot more difficult math and the conceptual clarity wasn’t there. Maybe there’s a different kind of conceptual clarity that would have been there since we wouldn’t be as stuck with visualising particles, literally moving on lines, which is what we’re invited to do when we think about things in terms of Feynman diagrams, even though it’s really, the quantum fields that are going on. So I don’t know if there would have been very big differences in how physicists think about the world. But anyway, I do think that the diagrams were gonna come at one point or another. 3:40:27.4 SC: Richard Cashdan says, “Suppose I ask a priority question today, and then during the next month, I run the universe splitter app a bigly number of times, to pretty much guarantee that I end up on a different timeline. Can I submit another priority question next month?” 3:40:43.8 SC: No, you cannot. ‘Cause I don’t know… What do you mean, end up on a different timeline? If you ask a priority question today, then all of the future versions of you are ones that descended from someone who asked a priority question today. You can’t undo the past by making measurements with the universe splitter or any other quantum mechanical measurement process. Sorry about that. If you could, I’d be writing papers about that, that’d be a very important thing. 3:41:11.7 SC: Okay, the very last question. This has been a long one today. Lewis B says, “How sure are we that things like Hawking radiation are literally true… Are likely true? My understanding is that the prediction of Hawking radiation rests on both quantum mechanics and general relativity being useful approximations in the vicinity of a black hole’s event horizon. But, isn’t the entire problem, those theories don’t play well together, in particular in regimes precisely like a black hole? Where, if anywhere, would you no longer trust these theories to be useful?” 3:41:44.1 SC: So again, there’s sort of a narrow version of this question and a broader one. The narrow version is, do we have good reasons to trust general relativity and quantum field theory in the event horizon of a black hole? And the answer is yes. So you’re saying, “Aren’t black holes exactly where we should not trust quantum field theory, or general relativity?” but that’s not quite true. The singularity near… In a future, when you get into a black hole, that’s where you shouldn’t trust general relativity. The general relativity says, all the equations are blowing up, the curvature is infinite, I have no reason to trust things anymore. But near the event horizon, the curvature of space time can be small, especially for a big black hole. So that’s precisely a regime in which you should trust general relativity perfectly. 3:42:39.3 SC: And in fact, here’s the tiny reason why you might be sceptical of that argument I just gave. So the argument I just gave was based on the idea that general relativity becomes untrustworthy when the curvature of space time becomes large, and that’s actually not true near the event horizon of a black hole, the curvature of space time can be small. 3:43:03.2 SC: In quantum field theory, there’s an analogous statement, “When do you trust quantum field theory?” Well, when it’s close to the vacuum state locally and you only have a few… A small number of particles. And again, near the event horizon of a black hole, you should be able to trust quantum field theory, it’s nearly the vacuum. If you fell into the black hole, you wouldn’t even notice, unless there are firewalls or something like that, in which case all bets are off. But let’s assume that there are not. 3:43:30.5 SC: The problem with what I just said, is exactly the black hole information loss puzzle. Namely, that both general relativity and quantum field theory, by all rights, should be trustworthy near the event horizon of a black hole. But taken together, they lead to the information loss puzzle. And so, that leads many people to say that somehow, in a subtle way, one of them is wrong, near the event horizon of a black hole, probably quantum field theory, in some subtle way. There’s some kind of non-locality that allows information to be transferred out of the outgoing event horizon. But… So even if that’s true, we think that if you looked at a black hole, the… Even if you think that information gets out in the radiation, you wouldn’t be able to tell whether radiation is getting out, just by looking at it. What you’d be doing is saying that the black hole is really analogous to classically setting a book on fire. If you classically just set a book on fire, let it burn, in principle, all of the information in the book is contained in the outgoing heat and light and ash and dust and so forth. But you can’t tell by looking at it, you can’t recover what was in the book once it’s burning. 3:44:53.1 SC: That would be the case, if you could actually look at an evaporating black hole. You would see the Hawking radiation, in principle, all the information is there, but in practice, it just looks like a black body, you can’t get it out. So that’s what we expect to have happen. Now, the other aspect of your question I wanted to raise here is, in principle, in general and other circumstances, how much confidence should we have in our theories when they’re being extrapolated into regimes far beyond where we’ve actually tested them? This is a very good question and a very hard one, and I don’t have a good answer for you. 3:45:38.1 SC: If it weren’t for black hole information loss, we’d be really pretty confident in quantum field theory in a wide variety of circumstances, and we think that black hole information loss is special, because gravity is so important in that case. So in practice, we are pretty confident in quantum field theory, in cases where gravity is not nearly black hole strength. That’s why I can say that the laws of physics underlying everyday life are very well understood right now, because we live in an environment which there’s no black holes nearby. So the usual reasons why we trust quantum field theory are likely to be valid, but maybe not. And then this is, again, something that we need to be cognisant of. 3:46:28.0 SC: I think, personally, that the extrapolations we do of known laws of physics are extremely likely to work when we get to black holes, because there’s nothing that breaks down, the things that seem to break down are things that we see exactly what is breaking down, the information getting out and in other cases, in other contexts, other questions we want wanna ask about it, it seems perfectly benign. But then there are other cases where we’re extrapolating into the early universe and there, people are a little bit sanguine, I think, about talking about ideas like inflation, or baryogenesis or other things that rely on extrapolations far beyond what we’ve tested in a laboratory, because they’re often not simply relying on robust features of quantum field theory, but they’re relying on specific features of specific quantum field theories. And those, I don’t have much faith in at all, when we’re extrapolating them to the very, very early universe. 3:47:31.0 SC: So, I think that in general, the physics that we currently established is so good, that we do become a little complacent about extrapolating it into regimes that we haven’t explicitly tested yet, and we should be a little bit more humble about that. That doesn’t mean we… All things go and anything can go and we shouldn’t have any faith at all, we should have a credence that we should keep under control, and the credence might be large, but it’s not 100%. So, this issue of where in general we should think our current favourite theories are valid or they’re applicable, the domain of applicability of a theory, that’s a good one that I think both physicists and philosophers should think harder about. We haven’t really nailed that one down, which makes it the perfect question to end on, something we don’t know the answer to. There you go. 3:48:23.9 SC: Okay, that was a lot of words. Thank you for sticking with me here. Thanks for asking the questions. Thank you for supporting the Mindscape podcast. I really do appreciate it. Come back with more questions next month. Bye bye.[/accordion-item][/accordion]

1 thought on “AMA | July 2021”

Comments are closed.

Scroll to Top